College Math 3
College Math 3
Level: B. Ed Programme
i
(All Rights are Reserved with the Publisher)
Quantity .........................................
ii
COURSE TEAM
iii
FOREWORD
Teaching and learning process started from the very beginning of human life in
this universe. Learning process needs effort of teachers for attainment of goals. It
largely depends upon the skills of the teachers which they attained during the pre-
service education. Mathematics most of the time considered difficult to
understand and it may be due to method and techniques of teaching that are less
interactive. Therefore there is a need that prospective teachers may be equipped
with content knowledge, skills and techniques for its teaching. This course aims to
involve and provide exposure to the prospective teachers to different
mathematical concepts. There is no exaggeration to highlight that mathematics
teaching is an integral and indispensible part of the teaching leaning process.
Therefore, for a teacher it is highly significant to understand the concepts of
mathematics and nature of instructional techniques. Teachers should be able to
plan and implement instructions in such a way that it provide mathematical
background of content and skill to teach these concepts. The development of this
course intends to strengthen the content knowledge of prospective teachers in
mathematics, this is third book of this series of mathematics content and focus on
important concepts relevant to plane curve, differential equation and Laplace
transform. The knowledge and skills gained during the course may help them
while practicing in the classroom and also assist to develop a more positive
attitude towards mathematics.
iv
INTRODUCTION TO COURSE
Mathematics has very close and basic functions to play in most of other subjects
such as Physics, Chemistry, and Astronomy and play pivotal role in development
of 21st centaury skills among the learners. The prospective teachers may be
competent in their concepts about mathematics, therefore major of mathematics is
introduced, and this is the third course of the series. We have already discussed
the basic concepts in the initial two courses. This particular course intends to
provide content knowledge competency of the prospective teachers. This course
comprises of conic section three dimensional geometry, differential equation and
Laplace transform. This course has been divided into four parts, the first part
involves concept of parabola, ellipse, hyperbola, tangent and normal, conics in
polar form, curve sketching and pedal equation, the second portion focuses on the
differential equations, solutions of differential equations, complementary solution
and general solutions of differential equations and third one carried the concept of
Laplace and inverse Laplace transform.
This course will equip the prospective teacher with knowledge and additional
skills to teach mathematics in grades 1 to grade 10th. This will enable the
prospective teacher to have a better concept of the subject and its techniques as
well.
The text includes relevant examples for the elaboration of the concepts and the
activities are placed for the hands on works, which consequently, help to enhance
content knowledge and the skills of the prospective teachers.
In the end, I am thankful to the Course Team, Editor and especially the unit
writers for quick and timely response and effort.
v
COURSE OBJECTIVES
After completing this course, the students will be able to:
1. Define parabola, circle, ellipse and hyperbola.
2. Differentiate between rotation of axis and translation of axis.
3. Explain the conics in term of polar coordinates.
4. Calculate problems regarding Cartesian coordinates
5. Describe the methods of finding angles between two curves.
6. Solve the problems of pedal equations.
7. Find out the angles between two lines and the planes.
8. Differentiate between the concept of direction ratios and direction cosines.
9. Differentiate between cylindrical and spherical coordinates.
10. Apply different techniques of solving ordinary differential equations.
11. Differentiate between the complementary solution and particular integrals.
12. Explain the concept of Laplace and inverse Laplace transform.
vi
CONTENT
Unit No. Unit Title Page No.
Unit -1 Plane Curve-I .......................................................................................01
Unit -2 Plane Curve-II ......................................................................................47
Unit -3 Plane Curve-III.....................................................................................79
Unit -4 Analytical Geometry of Three Dimensions-I ....................................123
Unit -5 Analytical Geometry of Three Dimensions-II ...................................157
Unit -6 First Order Differential Equations-I...................................................187
Unit -7 First Order Differential Equations-II .................................................221
Unit -8 Differential Equation of Higher Order ...............................................247
Unit -9 Laplace Transform .............................................................................285
vii
Unit – 1
PLANE CURVE-I
2
INTRODUCTION
The conic sections, or conics, are curves obtained by making sections, or cuts, at
particular angles through a cone. These curves were known to the ancient Greeks,
who first explored their properties. But, ancient as they are, these curves have
exceedingly modern applications — for example, the television dish aerial that
brings sports and films into our homes, and the giant radio telescopes such as the
one at Jodrell Bank in Cheshire, which look deep into space, all depend upon
what is known as the ‘reflective property’ of one of the conic sections.
If the cone is cut by a plane perpendicular to the axis of cone, then the section is a
circle.
The size of the circle depends on how near the plane is vertex of the cone. If the
plane passes through the vertex A, the intersection is just a single point or a point
circle. If the cutting plane is slightly tilted and cuts only one nappe of the cone,
the resulting section is an ellipse. If the intersecting plane is parallel to a generator
of the cone, but intersects is one nappe only, the curve of intersection is a
parabola. If the cutting plane is parallel to the axis of the cone and intersects both
of its nappes, then the curve of intersection is a hyperbola.
3
OBJECTIVES
After studying this unit you will be able to:
define conic sections.
describe various types of conic sections.
determine the standard form of the equations of the ellipse, the parabola and
the hyperbola.
graph of ellipse, parabola and hyperbola using its equations given in
standard from.
determine standard form for the equation of ellipse, parabola and
hyperbola given general form.
list and explain the formulas for each conic section.
find equations of the tangent line and the normal line and lengths of the
tangent and the normal between a point on a curve and the x axis for
various curves.
translate coordinate axes.
find new coordinates after translation.
find new equations if the origin is translated to a given point.
find new equation if the origin is rotated through given angle.
transform an equations by rotation of axes.
determine the difference between degenerate and non-degenerate conic
section.
4
1.1 Conic Section
The conic is the set of points in a plane, the distance of each of which from a
given point bears a constant ratio to its distance from a given straight line in the
plane.
A focus is a point about which the conic section is constructed. In other words, it
is a point about which rays reflected from the curve converge. A parabola has one
focus about which the shape is constructed; an ellipse and hyperbola have two.
A directrix is a line used to construct and define a conic section. The distance of
a directrix from a point on the conic section has a constant ratio to the distance
from that point to the focus. As with the focus, a parabola has one directrix, while
ellipses and hyperbolas have two.
The eccentricity of a conic section is defined to be the distance from any point on
the conic section to its focus, divided by the perpendicular distance from that
point to the nearest directrix. The value of e is constant for any conic section. This
property can be used as a general definition for conic sections. The value of e can
be used to determine the type of conic section as well:
If e=1, the conic is a parabola
If e<1, it is an ellipse
If e>1, it is a hyperbola
Note:
The eccentricity of a circle is zero. Two conic sections are similar (identically
shaped) if and only if they have the same eccentricity.
5
1.2 Parabola
A parabola is set of all points P in the plane that are equidistance from a fixed line
and a fixed point in the plane. The fixed point does not lie on the fixed line.
The fixed line is called the directrix of the parabola, and the fixed point is called
its focus. The straight line through the focus and perpendicular to the directrix is
called the axis of parabola. The point where the parabola meets its axis is called
the vertex of the parabola.
Figure 1.1
6
Which is standard equation of parabola.
Figure 1.2
Let the line through F(a,0) and perpendicular to the axis of the parabola meet the
parabola at L and . If is ordinate of L, then L(a, ) lies on
Case 1
If the equation of the parabola is ,a>0,
Then shape of this parabola is shown in figure 1.3
Figure 1.3
Case 2
If the equation of parabola is where a > 0.then shape of this parabola
is shown in figure 1.4
7
Figure 1.4
Case 3
If the equation of parabola is where a > 0.then shape of this parabola is
shown in figure 1.5
Figure 1.5
Case 4
If the equation of parabola is where a < 0.then shape of this parabola
is shown in figure 1.6
Activity
Analyze the parabola and draw its graph with
8
Solution
Here 4a = 12 or a=3
The focus of the parabola lies on the x-axis and opening of this parabola is to right
of y-axis.
Coordinates of the focus =(3,0).
Equation of axis is x=0
Length of the latusrectum is 12 and x=0 is tangent to the parabola at its vertex.
Activity
Analyze the parabola and sketch its graph
Solution
(1)
Let (2)
The equation (2) becomes (3)
Which is parabola whose focus lies on X=0 and whose directrix is thus
coordinates of the focus of (3) are:
Figure 1.8
Activity
Finds the focus and the directrix of the parabola
Solution
We find the value of a in the standard equation
so
Then we find the focus and directrix for this value of a:
Focus
Activity
Sketch the graphs of following equations:
i.
ii.
iii.
Solutions
i.
10
ii.
iii.
Activity
A concrete bridge is designed as a parabolic arch. The road over bridge is
40m long and the maximum height of the arch is 15m . Write the equation of the
parabolic arch.
Solution
From the graph the vertex is at (0, 0) and the parabola is open down
11
Equation of the parabola is x2 = −4ay
(−20, −15) and (20, −15) lie on the parabola
Therefore equation is
Activity
The equation y = (1/32) models cross sections of parabolic mirrors that are
used for solar energy. There is a heating tube located at the focus of each
parabola; how high is this tube located above the vertex of the parabola?
Solution
12
1.3 Ellipse
An ellipse is the set of points in a plane whose distances from two fixed points in the
plane have a constant sum. The two fixed points are the foci of the ellipse.
Figure 1.9
Is called an ellipse.
The number is eccentricity of the ellipse a focus and L a directrix.
13
Figure 1.10
14
Case 2
If the equation of the ellipse is , then the centre of the ellipse
is C(h,k) then shape is shown
Figure 1.12
Case 3
If the equation of the ellipse is then the centre of the
ellipse is C(h,k) and the shape is shown
Figure 1.13
Case 4
If the equation then the centre of the ellipse is C(h,k)
and the shape is shown
15
Figure 1.14
Activity
Find an equation of the ellipse having centre at (0,0), focus at (0,-3) and one vertex at
(0,4).sketch its graph
Solution Y
The second vertex has coordinates (0,-4)
Length of the semi major axis is
C=3 from we have
Figure 1.15
Activity
Find the centre, foci, vertices, centre ,semi minor axis, and semi major axis of ellipse
whose equation
Is given:
Activity
An arch in the shape of the upper half of an ellipse is used to support a bridge that
is to span a river 20 meter wide. The centre of the arch is 6 meters above the
centre of the river. Write an equation for the ellipse if the x-axis coincides with
the water level and the y-axis passes through the centre of the arch.
16
Solution
Let the x be along the base of the arch and the y-axis pass through its centre. An
equation of the ellipse representing the arch is:
Figure 1.16
Activity
The statuary Hall in the United States Capitol is elliptical. It measure 46 feet wide
and 96 feet long. If a person is standing at one focus, her whisper can be heard by
a person standing at the other focus. How far apart are the two people?
Solution
An equation of ellipse is
Figure 1.17
= 84.26ft
17
Activity
A semielliptical archway over a one-way road has a height of 3m and a width of
12m . The truck has a width of 3m and a height of 2.7m . Will the truck clear the
opening of the archway?
Figure 1.18
Solution
Since the truck’s width is 3m , to determine the clearance, we must find the height
of the archway 1.5m from the centre. If this height is 2.7m or less the truck will
not clear the archway.
From the diagram a = 6 and b = 3 yielding the equation of ellipse as
.
The edge of the 3m wide truck corresponds to x = 1.5m from centre We will find
the height of the archway 1.5m from the centre by substituting x = 1.5 and
solving for y
Thus the height of arch way 1.5m from the centre is approximately 2.90m . Since
the truck’s height is 2.7m , the truck will clear the archway.
1.4 Hyperbola
A hyperbola is the set of points in a plane whose distances from two fixed points in the
plane have a constant difference. The two fixed points are the foci of the hyperbola.
18
Figure 1.19
Figure 1.20
The asymptotes are drawn dashed as they are not part of the graph; they simply
indicate the end behavior of the graph. The equation of a hyperbola opening left
and right in standard form follows:
19
Figure 1.21
Here the center is (h,k) and the vertices are (h±a,k). The equation of a hyperbola
opening upward and downward in standard form follows:
Figure 1.22
The asymptotes are essential for determining the shape of any hyperbola. Given
standard form, the asymptotes are lines passing through the center (h,k) with
slope m=±ba. To easily sketch the asymptotes we make use of two special line
segments through the center using a and b. Given any hyperbola, the transverse
axis is the line segment formed by its vertices. The conjugate axis is the line
segment through the center perpendicular to the transverse axis as pictured below:
20
Figure 1.23
The rectangle defined by the transverse and conjugate axes is called the fundamental
rectangle
Activity
Discuss and sketch the graph of the equation:
Which is an equation of the hyperbola with transverse axis along the x-axis
Here a=4, b=5
From we have:
Figure 1.24
21
Activity
Find the centre, foci, vertices, eccentricity, asymptotes, equation of directrices of given
equation:
Solution:
Centre (0,0) Foci: Vertices: ( )
Eccentricity:
Equation of directrices :
Asymptotes :
Figure 1.25
Tangent:
If a line touches the curve at the point then this line is called tangent to
the curve at the point . The equation of the tangent at this point
to the curve is given by
Normal
A line passing through the point P and perpendicular to the L tangent at the point
P to the curve is called the Normal at that point P.
The equation of the normal at the point to the curve is
22
In the geometrical interpretation of the derivative of a curve or
that represents the slope of the tangent line to the curve . In
order to find an equation of the tangent to a given conic at some point on the
conic, we shall first find the slope of the tangent at the given point by calculating
from the equation of the conic at that point and then using the point slope form
of a lone it will be quite simple to write an equation of the tangent. Since the
normal to a curve at a point on the curve is perpendicular to the tangent through
the point of tangency, its equation can be easily written.
Activity
Find the equations of tangent and normal to the parabola x2 + 6x + 4 y + 5 = 0 at:
(1, −3).
23
Solution
Equation of parabola is
Equation of tangent is
Activity
Find the equation of normal to the parabola in the form:
Solution:
lies on
Put in (2)
24
Equation of normal
Is as required
Theorem
To show that a straight line cut a conic in a general, in two points and to find the
condition that the line be a tangent to the conic.
Proof
Let a line cut the conic:
(i)
(ii)
(iii)
(i) The point of intersection of
(1)
and (2)
Are obtained by solving (1) and (2) simultaneously for x and y. inserting the value
of y from (1) into (2), we get:
(3)
Which being quadratic in x gives two values of x. These values are the x
coordinates of the common points of (1) and (2).setting these values in (1), we
obtain the corresponding ordinates of the point of intersection. Thus the line (1)
cuts the parabola (2) in two points.
In order that (1) is a tangent to (2), the points of intersection of a line and the
parabola must be coincident. In this case, the roots of (3) should be real and equal.
This means that the discriminant of (3) is zero. Thus
(3)
Which is a quadratic in x and it gives the abscissas of the two points where (1)
and (2) intersect. The corresponding values of y are obtained by setting the values
of x obtained from (3) into (1). Thus (1) and (2) intersect in two points. Now (1)
is tangent to (2) if the point of intersection is a single point.
Hence
Are tangent to (2) for all values of
Activity
Find an equation of the tangent to the parabola which is parallel to the
line also find the point of tangency.
Solution:
Slope of the required tangent is m = -2
In the parabola (1)
26
Equation of tangent is
(2)
Inserting the value of y from (2) viz
Into (1) we have
Value
Putting this value of x in (2), we get
Activity
Find equation of the tangent of the ellipse
(1)
which is parallel to the line . Also find the point of contact
Solution:
The slope of the required tangent is .Equation of the tangent are
We solve (1) and (3) simultaneously to find the point of contact. Inserting the
value of y from (3) into (1), we get:
27
And so
Activity
Find equation of tangent and normal to each of the following curves at the
indicating point
at
Solution
The given equation is:
Diff w.r.t. x
Now
28
1.5.2 Intersection of Two Conics
It is known from algebra that the simultaneous solution set of two equations of the
second degree consists of four points. Therefore, two conics will always intersect
at four points. These points may all be real and distinct, two real and two
imaginary or all imaginary.
Activity
Find the intersection points of the ellipse and the
hyperbola .
Solution
The two equation may be written as
(1)
(2)
Adding Eq.(1) and (2) , we get
Activity
Find equations of the common tangent to the two conics;
=1 and
Solution
The tangents with slope m, to the two conics are respectively given by:
y = mx + b
and
For a tangent to be common, we must have;
29
1.6 Translation and Rotation of Axes
If in the plane with the given XX and YY axes new coordinate axes are chosen
parallel to the given ones, we say that there has been a translation of axes in the
plane.
Conversely, if the coordinates of a point with respect to the XY-system are given,
then the coordinates with respect to the original system can be determined by the
equations
30
Activity
Let P(8,3)and O′(2,–5)be two points in the XY-coordinates system. Find the XY-
coordinates of P referring to the translated axes O′X and O′Y.
Solution
Here . The coordinates of P referring to the
new XY-coordinates system are:
Activity
Let P (3,4) be a point referring to the XY-coordinate system translated
thorough O′(5,6). Find the coordinates of P referring to the original coordinate
system, the - System.
Solution
Here the coordinates of P referring to the new
XY-coordinates system are:
Activity
By transforming the equation (1)
Referred to a new origin and axes remaining parallel to the original axes, the first
degree terms are removed. Find the coordinates of the new origin and the
transformed equation.
Solution
Let the coordinates of the new origin be (h, k) equations of transformation are:
(2)
is to be so chosen that first degree terms removed from the transformed
equation.
Therefore, 2h-4=0 and 8k+8 = 0 giving h=2 and k= -1. New origin is
Into (2) the transformed equation is:
Activity
The point to which the origin is shifted and the transformed equation is given
below. Find the original equation?
x² + y² = 4 (3, -4),
31
Solution
Let the xy-coordinate system be rotated through an angle θ, such that the range of
the angle is about the same origin O. The new coordinate system
is XY-coordinate system as shown in the given diagram. Since triangle OBP is a
right triangle with
so,
– – (i)
32
Since (OB,BP) are the coordinates of the point P with respect to the new
coordinate system, XY-system, so .
Putting these values in (iii) and (iv), we have the following equations as:
These equations are used to find the coordinates of a point with respect to the new
rotated coordinate system, XY-system. Thus, the point P(x,y)with respect to XY-
plane is
Conversely, if the coordinates of a point with respect XY-system are given, then
its coordinates with respect to the original system can be determined by the
equations
Activity
The xy-coordinate axes are rotated about the origin through the angle of
measure . The new axes are OX and OY. Find the XY-coordinates of the
point P(1,6).
Solution
Here . The coordinates of P with respect to the
33
Activity
Find transformed equation of when the axes are rotated
through an angle .
Solution
The given original equation is:
Angle of rotation
35
There are several ways of classifying conic sections using the above general
equation with the help of the discriminant Δ of this equation:
or
Where
Now let
then the type of conic section that the above equation represents can be found
using the discriminant of the equation, which is given by for (1), or
If Δ=0:
the equation represents two distinct real lines.
36
If Δ 0:
, it represent hyperbola
Activity
Identify the conic for without rotating axes.
Solution
Let’s begin by determining A,B,C
A=5, B= C= 2
1. Find the focus, vertex and directrix of the parabola sketch its graph.
(ii)
(iii)
(iv)
(v)
4. A search light has a parabolic reflector (has a cross section that forms a
‘bowl’). The parabolic bowl is 40 cm wide from rim to rim and 30 cm deep.
The bulb is located at the focus .
(a) What is the equation of the parabola used for reflector?
(b) How far from the vertex is the bulb to be placed so that the maximum
distance covered?
ii.
iii.
6. Find an equation of ellipse with given data and sketch its graph.
i. Foci(0,-1) and (0,-5) and major axis of length 6
ii. Foci ( ) and passing through the point
iii. Vertices(0, ),eccentricity
iv. Centre (0,0), focus(0,-3), vertex (0,4)
v. Centre (0,0), major axis horizontal , the points (3,1),(4,0) lie on the graph
7. Find the centre ,foci, eccentricity, vertices and directrices of the ellipse
whose equation is given;
i.
ii.
iii.
iv.
v.
38
9. The maximum and minimum distances of the Earth from the Sun
respectively are 152 ×106 km and 94.5×106 km. The Sun is at one focus of
the elliptical orbit. Find the distance from the Sun to the other focus.
10. A narrow arch supporting a stone bridge is in the shape of half an ellipse and
is 24 meters long and 8 meters high. A person standing at one focus of the
ellipse throws a rubber ball against the arch, no matter what direction the
ball is thrown, it always bounces off the arch once and strike the same point
on the ground (the other focus). How far apart are the person throwing the
ball and the point on the ground at which the ball strikes?
i.
ii.
iii.
12. Find an equation of the hyperbola with the given data. sketch the graph
of each:
i. Foci Asymptotes
ii. Vertices asymptotes
iii. Centre (0,0)Focus (6,0) Vertex (4,0)
iv. Foci directrices
v. Centre (2,2), horizontal transverse axis of length 6 and eccentricity e=2
iv.
v.
14. Find equation of tangent and normal to each of the following curves at the
indicating point:
i. at (a,-2a)
39
ii. at
iii. at
15. Find equation of the tangent to each of the following through the given point:
i. through (7,-1)
ii. through (1,4)
16. Find equation of the normal to the parabola which are parallel to
the line:
17. Find equation of the normal to the ellipse which are parallel to
the line
22. Find an equation of each of the following with respect to new parallel axes
obtained by shifting the origin to the indicated point:
i.
ii.
iii.
iv. x + 3y = 6, (–4, 1)
40
23. When the origin is shifted to the point (2,3), the transformed equation of a curve
is find the original equation of the curve.
24. When the origin is shifted to the point (2,-1), the transformed equation of a
curve is find the original equation
of the curve.
25. Find coordinates of the new origin so that first degree terms are removed
from the transformed equation of each of following. Also find the
transformed equation.
26. When the axes are rotated through an angle , find the transformed equation
of
31. Find an equation referred to the new axes obtained by rotation of axes about
the origin through the given angle
41
2. (i)
(ii)
(iii)
(iv) arbitrary
(v)
3. 4 m
4. (a) y2 = 40/3 x.
(b) 10/3 cm
5. (i)
(ii)
42
(iii)
(i)
(ii)
(iii)
(iv)
(v)
(ii)
(iii)
44
12. Sketch the graphs yourselves
(i)
(ii)
(iii)
(iv)
(v)
Directrices:
Directrices:
(v) Centre: (2, ), Foci: (2, ), vertices: (
eccentricity: ,
Directrices:
iii.
21.
22. (i)
(ii)
(iii)
(iv) X + 3Y = 7.
23. (
24.
25.
26.
27.
28. Hyperbola
29.
30.
31.
46
Unit – 2
47
CONTENT
Introduction ...................................................................................................49
Objectives .....................................................................................................49
2.1 Rectangular Co-ordinates System .................................................................50
2.2 Polar Co-ordinates ........................................................................................52
2.3 The Polar Equation for a Conics ...................................................................57
2.4 Graphs of Polar Equation ..............................................................................59
2.5 Conic Sections in Polar Coordinates.............................................................60
2.6 Identifying a Conic in Polar Form ................................................................60
2.7 The Polar Equation for a Conic ....................................................................61
2.8 Graphs of Polar Equations ............................................................................64
2.9 Exercise and Solutions ..................................................................................75
Further Readings ...........................................................................................78
48
INTRODUCTION
This unit is based on the idea of plane curve and focus upon the rectangular and
polar coordinate system. We introduced polar coordinate system and provided the
example to solve the problem related to polar and rectangular coordinates.
Application of coordinate system is focused in this unit to strengthen the skill and
knowledge of the prospective teachers. Conic section in polar coordinates is the
important concept to understand, it is elaborated by providing examples and
activities. Different polar equations, its solutions and graphs have also been
discussed in this unit.
OBJECTIVES
After studying this unit, prospective teachers will be able to:
1. explain the concept of rectangular coordinate system.
2. describe polar coordinate system.
3. solve the equations given in polar coordinate system
4. draw the graphs of polar equations.
49
2.1 Rectangular Coordinate System
Figure 2.5
Solution:
In this example, the points appear to
be collinear, or to lie on the same line.
The entire chapter focuses on finding and
expressing points with this property.
Figure 2.6
51
2.2 Definition of Polar Coordinates
Before we can start working with polar
coordinates, we must define what we will
be talking about. So let us first set us a
diagram that will help us understand what
we are talking about. First, fix an origin
(called the pole) and an initial ray from O.
Each point P can be located by assigning
to it a polar coordinate pair (r, ) where r
is the directed distance from O to P and
is the directed angle from the initial ray
to ray OP. When is positive, then the
angle was measured counterclockwise, Figure 2.7
and when is negative, the angle was
measured clockwise. By this fact, a
given polar coordinate is not unique.
2. Draw a line through the origin that makes an angle θ with the +ve x-axis.
The angle is positive in the counter clockwise direction and negative in the
clockwise direction. Call this line L:
52
Figure 2.9
3. Imagine rotating the x-axis through the same angle and making it coincide
with the line L. Mark as positive the part of the line L that the positive x-
axis coincides with and mark as negative the part that the negative x-axis
coincides with. This is similar to what we did in the x,y coordinate system:
Figure 2.10
4. Find the point on L that is a distance r from the origin. Positive and negative
distances are in those parts of L that we marked positive and negative above
(figure 10). The point that you marked is the point that corresponds to (r, θ)
in the polar coordinate system.
Figure 2.11
53
That’s it. That’s the rule for polar coordinates. The numbers (r, θ) are called the
polar coordinates of the point we plotted.
Examples:
Below are some examples of plotting points using their polar coordinates. Try to
do the examples yourself and compare the results. Keep in mind that all angles are
in radians. Be sure that you can do and understand the examples c-d (Hint: 13π/6
= 2π + π/6).
Figure 2.12
54
for n positive and clockwise for n negative. Hence, the lines L
corresponding to θ1 and θ2 = θ1 ± 2πn are the same and have the same
positive and negative parts. Going the same distance r1 = r2 gets to the
same point.
Figure 2.13
(b) (This may be little difficult to understand the first time.) It can happen
if r2 = −r1 and θ2 = θ1 ± π ± 2πn for any non zero integer n (positive
or negative). The lines corresponding to θ1 and θ2 = θ1 ± π (and hence
θ2 = θ1±π±2πn) have the same inclination but the positive and
negative parts are switched. Therefore going the distance r2 = −r1
gives the same point. The following figure shows this. Be sure that you
understand the signs on the line L.
Figure 2.14
2. All polar coordinates (0, θ) map to the same point. This is so because for
any θ the point that is distance 0 away from the origin along the line L is the
origin:
55
Figure 2.15
56
2. Restrict θ to one of the ranges
0 ≤ θ < 2π, or,
0 < θ ≤ 2π, or,
−2π ≤ θ < 0, or,
−2π < θ ≤ 0.
and restrict r to nonnegative values (i.e. r ≥ 0). Again draw a sketch
and see that this works. As I said above, fixing the problem 2
described in the previous section is impossible if we want polar
coordinates for the origin.
With this discussion of restricted polar coordinates, we have finished
the first point of this note – to understand what the polar coordinate
system is.
Figure 2.16
First, let’s hold θ fixed, and vary r. This just gives us the line L. Then, we change θ,
fix it again, and vary r. This gives another line L. Thus, holding θ fixed at different
values and varying r gives us a pattern of rays spreading out from the origin:
Figure 2.17
57
Next, let’s hold r fixed and vary θ. This gives a circle of radius r with the origin as
the center:
Figure 2.18
Figure 2.19
Changing the values of r and repeating this procedure gives a set of concentric
circles:
Figure 2.20
58
Putting the rays and the concentric circles together we get the polar coordinate grid:
Note that the circles collapse to a point at the origin. This is really just the visual
manifestation of the fact that the point (0, θ) is the origin for any θ. The collapse
of the circle to a point at the origin turns out to be a serious problem in many
applications of polar coordinates and you should forever be alert to this issue.
Figure 2.21
Because OAB is a right angled triangle (why?), it is easy to express the x and y
coordinates (which we denote u and v respectively) in terms of the r and θ coordinates:
u = r cos θ
v = r sin θ.
Similarly,
59
Thus, we can write r and θ in terms of x and y as:
That’s all there is to the relation between the two coordinate systems.
In an elliptical orbit, the periapsis is the point at which the two objects are
closest, and the apoapsis is the point at which they are farthest apart. Generally,
the velocity of the orbiting body tends to increase as it approaches the periapsis
and decrease as it approaches the apoapsis. Some objects reach an escape
velocity, which results in an infinite orbit. These bodies exhibit either a parabolic
or a hyperbolic orbit about a body; the orbiting body breaks free of the celestial
body’s gravitational pull and fires off into space. Each of these orbits can be
modeled by a conic section in the polar coordinate system.
60
Figure 2.22
We previously learned how a parabola is defined by the focus (a fixed point) and
the directrix (a fixed line). In this section, we will learn how to define any conic in
the polar coordinate system in terms of a fixed point, the focus P(r,θ) at the pole,
and a line, the directrix, which is perpendicular to the polar axis.
If F is a fixed point, the focus, and D is a fixed line, the directrix, then we can
let ebe a fixed positive number, called the eccentricity, which we can define as
the ratio of the distances from a point on the graph to the focus and the point on
the graph to the directrix. Then the set of all points P such that e=PF/PD is a
conic. In other words, we can define a conic as the set of all points Pwith the
property that the ratio of the distance from P to F to the distance from P to D is
equal to the constant e.
With this definition, we may now define a conic in terms of the directrix, x=±p, the
eccentricity e, and the angle θ. Thus, each conic may be written as a polar equation, an
equation written in terms of r and θ.
For a conic with a focus at the origin, if the directrix is x=±p.where p is a positive
real number,
61
and the eccentricity is a positive real number e, the conic has a polar equation
r=ep/ ( 1±ecosθ)
For a conic with a focus at the origin, if the directrix is y=±p, where p is a
positive real number, and the eccentricity is a positive real number e, the conic
has a polar equation.
r=ep / ( 1±esinθ)
How to: Given the Polar Equation for A Conic, Identify The Type of Conic,
The Directrix, and The Eccentricity
Multiply the numerator and denominator by the reciprocal of the constant in the
denominator to rewrite the equation in standard form.
Example:
For each of the following equations, identify the conic with focus at the origin, the
directrix, and the eccentricity.
Solution:
For each of the three conics, we will rewrite the equation in standard form.
Standard form has a 1 as the constant in the denominator. Therefore, in all three
parts, the first step will be to multiply the numerator and denominator by the
reciprocal of the constant of the original equation, 1/c, where c is that constant.
a. Multiply the numerator and denominator by 1/3.
62
Because sinθ is in the denominator, the directrix is y=p. Comparing to standard form,
note that e=23.Therefore, from the numerator
Since e<1, the conic is an ellipse. The eccentricity is e=23 and the directrix
is y=3.
63
Since e>1, the conic is a hyperbola. The eccentricity is e=5/4 and the directrix
is x=12/5=2.4.
c. Multiply the numerator and denominator by 1/2۔
Because e=1, the conic is a parabola. The eccentricity is e=1 and the directrix
is y=−7/2=−3.5.
Self Exercise
Identify the conic with focus at the origin, the directrix, and the eccentricity for
r=2/ (3−cosθ).
Answer
ellipse; e=1/3 and x=−2.
64
coordinate grid) and five different lines passing through the pole to represent the
angles at which the exact values are known for the trigonometric functions.
Figure 2.23
65
Figure 2.24
Note: It is possible for a polar equation to fail a test and still exhibit that type of
symmetry when you finish graphing the function over a full period. When you
started to graph functions (in rectangular form) you stared by learning the basic
shapes of certain functions such as lines, parabolas, circles, square roots, and
absolute value functions just to name a few. Polar equations also have some
general types of equations. Learning to recognize the formulas of these equations
will help in sketching the graphs.
Figure 2.25
The limacons containing sine will be above the horizontal axis if the sign between
a and b is plus or below the horizontal axis if the sign if minus. If the limaçon
contains the function cosine then the graph will be either to the right of the
vertical axis if the sign is plus or to the left if the sign is minus.
66
The ratio of a b will determine the exact shape of the limaçon
a/ b < 1 a/ b = 1 1 < a/ b < 2 a/ b ≥ 2
Figure 2.26
The graphs of limaçons with cosine would have similar shapes but along the
horizontal axis.
d) Rose Curves
A rose curve is a graph that is produced from a polar equation in the form of:
r = a sin nθ or r = a cos nθ, where a ≠ 0 and n is an integer > 1
They are called rose curves because the loops that are formed resemble petals.
The number of petals that are present will depend on the value of n. The value of
a will determine the length of the petals.
67
Figure 2.27
Figure 2.28
e) Lemniscates
The last type of polar equation that we will cover here is the lemniscates,
which has the shape of a figure-8 or a propeller. Lemniscates have the
general polar equation of:
r²= a² sin 2θ or r² = a² cos 2θ, where a ≠ 0
A lemniscate containing the sine function will be symmetric to the pole while the
lemniscate containing the cosine function will be symmetric to the polar axis, to θ
= π/2, and the pole.
Example:
Graph the polar equation r = 1 – 2 cos θ.
68
Solution:
Identify the type of polar equation
Figure 2.29
Since the ratio is less than 1, it will have both an inner and outer loop. The loops
will be along the polar axis since the function is cosine and will loop to the left
since the sign between a and b is minus.
Evaluate r at different values of θ Since the equation passes the test for symmetry
to the polar axis, we only need to evaluate the equation over the interval [0, π] and
then reflect the graph about the polar axis.
69
70
Example 1 (Continued):
Figure 2.30
71
Example 2:
Graph the polar equation r = 3 cos 2θ.
Solution:
Identify the type of polar equation
The polar equation is in the form of a rose curve, r = a cos nθ. Since n is an even
integer, the rose will have 2n petals.
2n = 2(2) = 4 petals
Test for symmetry
72
Example 2 (Continued):
Evaluate r at different values of θ
73
Example 2 (Continued):
Figure 2.31
These points will provide us with enough points to complete the rest of the graph
using the symmetry of the rose curve.
Figure 2.32
Use the symmetry to complete the graph
74
2.9 Exercise and Solutions
1. Convert the conic to rectangular form.
Solution:
We will rearrange the formula to use the identities
Solution
3. Find the polar form of a conic given a focus at the origin, e=3/5, and
directrix x=4.
Solution:
Because the directrix is x=p, we know the function in the denominator is cosine.
Because x=4, 4>0, so we know there is an addition sign in the denominator. We
use the standard form of
75
Therefore
4. Find the polar form of the conic given a focus at the origin, e=1, and
directrix x=−1.
Solution:
5. Find the polar form of the conic given a focus at the origin, e=3 and
directrix y=−2.
Solution
The directrix is y=−py=−p, so we know the trigonometric function in the
denominator is sine.
Because y=−2, –2<0, so we know there is a subtraction sign in the denominator.
We use the standard form of
76
Therefore
6. Draw graph
Solution
Figure 2.33
Figure 2.34
77
Solution:
Solution
Basically, what we need to do here is to convert all the x’s and y’s into r’s and θ’s
using the following formulas.
Don’t forget about the last one! If it is possible to use this formula (and you can
see where we’ll use it in the case can’t you?) it will save a lot of work!
First let’s substitute in the equations as needed.
Further Readings
1. C.h. Edward and E.D Penney, Calculus and Analytic Geometry, Prentice
Hall, Inc, 1988.
3. H.Anton, I.Bevens, S.Davis, Calculus, 8th Edition, John Wiley and Sons,
In. 2005.
78
Unit – 3
79
CONTENT
Introduction ...................................................................................................81
Objectives .....................................................................................................81
3.1 Asymptotes ...................................................................................................82
3.2 Maxima and Minima .....................................................................................90
3.2.1 Concavity Definition.........................................................................93
3.2.2 Points of Inflection ............................................................................95
3.2.3 Second Derivative Test for Extrema .................................................96
3.3 Sketching the Curves in Cartesian Coordinates ..........................................106
3.4 Tangents and Normal ..................................................................................113
3.4.1 Tangents and Normal (Polar Coordinates) .....................................116
3.4.2 Measure of the Angle of Intersection of two Curves ......................118
80
INTRODUCTION
We have already learnt in unit II, the method of drawing graphs of conics. For
sketching curves, information about asymptotes, critical points (maxima,
minima), concavity and singular points etc, are focused in this unit. For better
accuracy of sketching curves, it will be helpful to know about these topics.
OBJECTIVES
After studying this unit, prospective teachers will be able to:
1. sketch different types of curves.
2. find angle of intersection of curves.
3. solve the pedal equations
4. find the equation of tangents to curve.
5. find the equations of normal to curves.
81
3.1 Asymptotes
Definition: A straight line l is called an asymptote for a curve C if the distance
between l and C approaches zero as the distance moved along l (from some fixed
point on l) tends to infinity.
The curve C can approach asymptote as Ɩ one moves along Ɩ in one section, or in
the other direction, or in both directions.
Suppose the equation of C is such that y is real and as if
Similarly, if as or then is an
one side then straight line is a vertical asymptote. To see this observe that
(i) is a distance between the curve and straight line and that this distance
82
And consider those values of y for which θ(y) = 0. Similarly, to find a vertical
asymptote, we write the given equation in the form of and consider those
Example 1
Consider
Figure: 3.1
Example 2
Consider
As thus, x = 0, is an asymptote.
Example 3
83
Here,
Thus as or ,
We find that as
Example 4
Consider the curve x3y2 + 3x2y2 – (x – 2)3 = 0
The given equation can be written in the form.
Thus, x = -2 is an asymptote. For 0 < x < 2, the numerator in (1) is negative and
the denominator is positive. Thus y is not real. Similarly, for -2 < x <0, the
numerator is negative and the denominator is positive. Thus, y is again not real.
Hence as from either side, y is not real and so x = 0 is not an asymptote.
Example 5
Consider the curve 2xy + 2y = (x – 2)2
Here,
As ,| . Therefore, x = -1 is an asymptote.
84
We find that as . Thus there is no horizontal asymptote.
Now suppose that y = mx+c is an inclined asymptote of the given curve. The
abscissa of the points of intersection of the curve and the line are given by
2 x (mx + c) + 2 (mx + c) = (x – 2)2
X2(2m – 1) + x (2c + 2m + 4) + 2c – 4 =0------------------------(1)
Dividing by x2 and letting , we have
2m – 1 = 0
Or
2c + 5 =0
So that c = - 5/2.
Thus, is an asymptote of the curve.
Draw tangent and normal lines at P. Through the pole O draw a line perpendicular
to the radius vector meeting the
85
Figure No. 3.2
Let the curve have an asymptote . Then the radius vector OP. Let the curve and
The equation of the other asymptotes corresponding to the direction , etc. can
be found in a similar manner.
Or,
Now,
i.e a = r cos
and a = - r cos
are the required equations of the asymptotes
87
Example 8
Find the asymptotes of the curve
.. r = a sec + b tan
Solution
The given curve is
Or
as etc. Thus there are only two distinct asymptotes. Their equations
are
88
i.e the asymptotes are
a+ b = r cos
a – b = r cos
Exercise 3.1
Find the asymptotes of the curve with the following equations.
1.
2. x2y2 = x2 +3
3. 2xy = x2 + 3
4. x2(x – y)2 +a2 (x2 – y2) = a2xy
5. (x – y)2 (x2 +y2) – 10 (x – y) x2 + 12 y2 + 2x + y =0
6. x2y+xy2+xy+y2 + 3x = 0
7. (x – y + 1) (x – y – 2) (x+y) = 8x – 1
8. y3 + x2y + 2xy2 –y4 +1 = 0
9.
10.
11.
12.
13.
14.
15.
89
3.2 Maxima and Minima (of a Function of One Variable)
Definition
Let a function be defined on [a, b]. is said to have an absolute maximum at a
point c belonging to [a, b] if ( c) ≥ (x) for all x ∊ [a, b]. The number (c) and
(d) are called respectively absolute maximum and absolute minimum values of
on [a, b].
The term (relative) extreme value (extrema) is used to refer to either a (relative)
maximum value of a (relative) minimum value.
90
and
--------- (1)
--------- (2)
The case when has a relative minimum at c can be handled in a similar manner.
Note: The condition/ (c) = 0 is necessary but not sufficient. If / (c) = 0 we can
only conclude the y = (x) has horizontal tangent at x = c; it may or may not have
a relative extrema there. From y = (x) = x3 has / (c) = 0. But (0,0) is not a
maximum or minimum point. [see fig..]
– h, c + h], h<0 , except possibly at x =c. then has a relative extreme value at
x = c if changes sign at x = c, i.e. for all x = c. if changes sign at x =
Proof
We prove (ii), since < 0 fro x < c, is a decreasing function for x < c.
therefore, (x) ≥ (c) for all x ∊ [c, c+h]. Hence for all x ∊ [c – h, c+h] we have
(x) ≥ (c) for all x ∊ [c, c+h]. hence for all x ∊ [c – h, c +h] we have
(x) ≥ (c)
It follows from definition that has a relative minimum value at x = c.
Example 9
Find the relative extreme value of
(x) = 4x4 – 8x3 + 22x2 – 24x + 1
92
=0 when x = 1,2,3
<0 if x < 1
>0 if x = 3
Figure 3.5
Figure 3.6
93
We know that if > 0 in some interval, then is an increasing function.
Then as a point P moves left to right along the curve (Fig….), the slope of the
curve increases. Thus the curve is concave up.
Similarly, if < 0 in an interval, then is a decreasing function. In this
case as a point P moves from left to right along the curve, the slope of the curve
decreases (Fig….) the curve is concave down under such circumstances.
to [a, b].
ii. y = (x) is concave down in [a, b] if and only if < 0 for all x
Solution
For x > 2,
Thus the curve faces up in the intervals x < 1, x > 2 and face down in [1, 2]
94
3.2.2 Points of Inflection
The tangent line to a curve at a point of inflection P always crosses the curve
because the curve must face up on one side of P and be above the tangent there,
while the curve must face down and be below the tangent on the other side.
Figure 3.7
The curve y = (x) changes the direction at a point of inflection P. Hence, if is
Note: the converse of the above theorem is not true. It does not always happen
that x = c is a point of inflection when =0.
Now (0,0) is not a point of inflection, for the graph faces up for x > 0 and also for
x < 0.
95
3.2.3 Second Derivative Test for Extrema
Theorem :- if y = (x) has a second order derivative in an interval that
includes x = c as interior point with = 0 and < 0 (or >0) in this
Example 12
Let y = (x) = x4 – 8x3 + 22x2 – 24x + 1
= 4 (x3 – 6x2 + 11x – 6)
= 4(x – 1) (x – 2) (x – 3)
And = 0 for x = 1, 2, 3.
If x = 1 = 4 (3 – 12 + 11) > 0
Example 13
Find the point of inflection of the curve.
Solution
96
When x = 6, =0
And when x = 6,
Example: 14
Show that the height of an open cylinder of given surface s and greatest volume is
equal to the radius of its base.
Solution: Let r be the radius of the circular base, h the height, S the surface and V
the volume of the open cylinder, Then.
(1)
And (2)
97
Thus r varies in the interval (0,
Now,
maximum at r =
For
Exercise 3.2
1. Locate the points of relative extrema of
(x) = 2x3 – 15x2 + 36x + 10 (ii) (x)=3x4 – 4x3 + 5
2. Determine the values of x for which the function
(x) = 12x6 – 45x4 + 40x3 + 6
3. Find the points of relative maximum for
(x)=(x – 1) (x – 2) (x – 3)
4. Find the points of relative maximum, relative minimum, absolute maximum,
absolute minimum for the functions.
y = x3 – 12x2 + 45x in the interval [0,7]
5. Show that (x)= xx has a relative minimum for x = e-1
98
6. Find the maxima and minima for the radii vector of the curve
Singular points
Figure 3.8
A point through which there pass two branches of a curve is called a double
point.
99
A point through which, pass r branches of a curve is called a multiple point. A
multiple point is also called a singular point.
A curve has two tangents at a double point, one for each branch.
The double point is called a Node, a cusp or an isolated point according as the
two tangents are different and real, coincidently or imaginary.
Figure 3.9
Figure 3.10
Figure 3.11
The above Figure is the sketch of the cure ay2 – x (x + a)2 = 0. Here A (-a, 0) is a
double point of the curve and is an isolated point. The two branches of the curve
have imaginary tangents there.
100
If P (x, y) is any point on the curve, the slope of the line
When x →0 we have
b1 + b2m = 0
if b2 ≠ 0, m = - Hence, i.e, b1x + b2y = 0 is a tangent at the origin.
In general, the equation of the tangent (or tangents) at the origin is obtained by
equating to zero the terms of the lowest degree in the equation of the curve.
Example15
The tangents at the origin to the curve
For the curve x3 + y3 – 3axy = 0 the tangents at the origin are x = 0, y = 0. Hence
the origin is a node.
Example 16. For the curve (x2+y2) x – 2ay2 = 0, tangent at the origin is y = 0
and so the origin is cusp.
101
Example 17.
a2 x2 + b2y2 = (x2+y2)3 ,
are ax + ib y = 0, which are imaginary. So the origin is an isolated point.
(x + 1 – y – 2 )2
(x – y – 1 )2 = 0
i.e., x – y – 1 = 0 or y = x – 1 is a pair of coincident tangents. Thus (-1, -2) is a
cusp.
Example 19. Find the multiple points on the curve.
x4 – 2ay3 – 3a2y2 – 2a2x2 + a4 = 0
solution. Differentiating the given equation w.r.t. x we have.
Or,
( -6 ay2 – 6a2y) = 4a2 – 4x3
Or,
-6ay (y + a) = 4x (a2 – x2) (1)
At a multiple point, (1) being an equation of the first degree is satisfied by more
than one value of Hence y(y + a) = 0, 4x (a2 – x2) = 0. Thus, solution set of
102
{(a, 0,) (-a, 0)(0, -a) lie on the curve.
Hence they are the required multiple points.
Figure 3.12
In Fig. 6.9 (a), the two branches lie on the same side of the common normal and
on 'the different sides of the tangent. In Fig. 6.9 (h), the two branches lie on the
same side of the normal and on the same side of the tangent. In Fig. 6.9 (c), the
two branches lie on the different sides of the normal and on the different sides of
the tangent. In Fig. 6.9 (d), the two branches lie on the different sides of the
normal and on the same side of the tangent. In Fig. 6.9 (e) the two branches lie
on the normal but on one side they lie on the same and on the other on opposite
sides of the common tangent. One branch has inflection at the point.
A cusp is single or double according as the two branches lie on the same or different
sides of the common normal. Also it is of the first or second species according as the
branches lie on the different or on the same side of the common tangent.
103
Example 20. Find the nature of the cusp on the curve y2 = x3
Solution. y = 0 is the tangent at the origin.
Since x cannot be negative, the two branches lie on the same side of the common
normal. Thus the cusp is single.
Since y = ± x3/2, to each positive value of x correspond two values of y which are
of opposite signs and hence the two branches lie on different sides of the
common tangent and the cusp is of first species.
i.e.,
(1)
Equation (2) are necessary and sufficient conditions for any point (x,y) on f(x,y)
=0 to be multiple point.
To find multiple points (x,y), we find the values of (x,y) which simultaneously
satisfy the three equations.
So that at the multiple point, where then value of are the roots of
104
In case are not all zero and , the point (x,y) will be
double point and will be a node, cusp or an isolated point according as the values
of are real and distinct, equal or imaginary i.e. according as
the second.
Now
Now
At (0,0),
Exercise 3.3
1. Find whether the origin is a node or a cusp.
(i) (x2 + y2)2 = 4a2 xy (ii) y2 (a2 – x2) = x2 (b – x)2
(iii) (x2 + y2) (2a – x) = b2x (iv) a2 (x2 – y2) = x2y2
2. Show that the origin is a node, a cusp or an isolated point on the curve y2 =
ax2 + ax3 according as a is positive zero or negative.
3. Find the position and nature of the multiple points on the following curves.
(i) x2 ( x – y) + y2 = 0
(ii) y3 = x3 + ax2
(iii) x4 + y3 – 2x3 + 3y2 = 0
105
4. Fin the equation of the tangents at the multiple points of the following
curves:
(i) x4 – 4ax3 – 2ay3 + 4a2x2 + 3a2y2 – a4 = 0
(ii) (y – 2)2 = x (x – 1)2
Symmetry
(i) If , the curve is symmetrical about the x-axis in this case,
Figure 3.13
(ii) If the curve is symmetrical about the y-axis. This is
Figure 3.14
106
(iii) If , the curve is symmetrical about the line y = x
Figure 3.15
(iv) If there is symmetry in the opposite quadrants, the
Figure 3.16
i. Find out if the origin lies on the curve. If its does, write down the tangents
thereat. In case the origin is a multiple point, find out its nature.
ii. Find out the intercepts of the curve on the axes. These points are found by
setting y and x equal to 0 in the equation and solving for the other
coordinate.
iii. Find out and the points where the tangent is parallel to the coordinate axes.
107
Example 21. Sketch the curve
y2 (a2 + x2) = x2 (a2 – x2)
Solution. We note the following particulars about this curve.
i. It is symmetrical about the both axes.
ii. It passes through the origin and y = + are the two tangents there at. Thus
the origin is a node.
iii. It means x-axis at (a,0), (0,0) and (-a, 0) and meets y-axis at (0,0) only.
The tangents at (a,0) are x = a and x = - a.
iv.
when x =
v. It has no asymptote.
vi. As is real when –a ≤ x ≤ a.
viii. When x = 0, y = 0
Figure 3.17
At x =
again as x
108
Increases from to a, y decreases and y = 0 when x = a. The sketch of
is symmetries about the pole. In such a case, the pole is the centre of the
curve.
vii. Position of the Pole relative to the Curve
viii. See whether the pole lies on the curve by putting in the equation of
109
iii. When
Figure 3.18
and r is not real. Thus, no point on the curve corresponds to these values of
. When = , r = 0
110
Figure 3.19
Example 20. Sketch the curve
unchanged. Hence the graph is symmetrical about the line perpendicular to the
initial into. The graph can be easily sketched as show below.
Figure 3.20
Example 21. To Sketch the Spiral of Archimedes
Solution:
i. when
Also when
When
Thus, the curve starting from the pole goes round in both ways an infinite number
of times. The continuously drawn line corresponds to positive value of and the
dotted one to negative values of .
111
Figure 3.21
Now, sin
Figure 3.22
Solution. When = 0, r = a
When increases, r also increases.
Also when ⟶∞, r ⟶∞, when ⟶-∞, r ⟶ 0
Again r is always positive. The graph is as follows:
112
Figure 3.23
Exercise 3.5
2.
3.
5. (Lemniseate)
6.
7.
8.
given by
113
If we write to denote , the first equation can be written in the
form
and
r2 = x12 + y12
Example: Show that the equation of the tangent to the conic
at the point (x1,y1) can be written in
the form
114
iii) The hyperbola
Or,
Or,
Therefore,
i.e.,
or,
or,
115
Example: Find the equation of a normal to the parabola in the form
Let
Since
= mx + am3
i.e. y = mx – 2am – am3
Figure 3.24
116
Or,
Or,
Dividing by we get
As
And
Or,
between the radius vector and the tangent vector at P in the direction of
117
3.4.2 Measure of the Angle of Intersection of two Curves
angels between the vector and the tangents to the two curves at P, then
Figure 3.25
Let OL be the perpendicular from O to the tangent at to the curve defined
by
Let
Or,
Now,
118
Thus,
From
….2
between
And
Example 24. Show that the radius vector is inclined at a constant angel to the
tangent at any point on the equi-angular spiral
Or,
which is constant.
119
Example 25. Find the angle of intersection of the cardioids
angles which makes with the tangents to the two curves. For the curves
Therefore,
Or,
Or,
Therefore,
Thus,
Solution. Here
120
Or,
Or,
Therefore, tan
Or,
Now,
Thus,
Or,
Exercise 3.6
1. Find the equation of the tangent and normal to each of the following curves:
121
4. Find the measures of angle between the curves
(i) (ii)
(iii) (iv)
5. Show that the tangents to the cardioids r = a (a cosG) at the points with
and are respectively parallel and perpendicular to the initial line.
6. Show that
(iii) (iv)
(vii) (viii)
measure of the angle between the radius vector and the outward pointed
normal is 2 0.
9. At any point (r, ) of a polar curve measure of the angle between the radius
vector and the tangent vector in the direction of increasing 9 is show that the
curve is a cardioid.
122
Unit – 4
ANALYTICAL GEOMETRY OF
THREE DIMENSION-I
123
CONTENT
Introduction .................................................................................................125
Objectives ...................................................................................................125
4.1 Rectangular Coordinates ............................................................................126
4.1.1 The Three-Dimensional Coordinate System ...................................126
4.2 The Distance and Midpoint Formulas ........................................................128
4.2.1 Distance Formula in Space ............................................................129
4.3 Vectors in Three dimensional coordinate System ......................................130
4.3.1 Vectors in 3D .................................................................................131
4.3.2 Angle Between Two Vectors ..........................................................132
4.3.3 Parallel Vectors ...............................................................................133
4.3.4 Direction Cosines ...........................................................................134
4.4 The Cross Product .......................................................................................135
4.4.1 Scalar Triple Product ......................................................................138
4.4.2 Lines in Space .................................................................................138
4.4.3 Plane in space .................................................................................140
4.5 Angle Between two Planes .........................................................................143
4.5.1 Sketching Planes .............................................................................145
4.5.2 Distance Between plane and a point ..............................................147
4.6 Summary .....................................................................................................151
Further Readings .........................................................................................156
124
INTRODUCTION
Within classical mathematics, analytical geometry is the study of geometry using
a coordinate system, also known as coordinate geometry and cartesian geometry.
Analytic geometry is used widely in physics and engineering, as well as in
aviation, space science, and spaceflight. It is the basis of most modern geometry
fields, including algebraic, differential, discrete, and computational geometry.
Typically the Cartesian coordinate system is used to handle plane, straight and
square equations, often in two and sometimes in three dimensions. Geometrically,
the Euclidean plane (two dimensions) and the Euclidean (three dimensions) are
studied. Analytical geometry, is dealing with numerically identifying and
representing geometric shapes and extracting numerical knowledge from
numerical definitions and representations of shapes.
OBJECTIVES
After studying this unit you will be able to
1. Specify a point in three dimensional space and describe the three
dimensional coordinate system.
2. Apply distance and midpoint formulas in space for two points.
3. Describe various vectors in three dimensional space.
4. Determine the angle between two planes by selecting one point or plane.
5. Discuss different dimensions of cross product and scalar triple products.
6. Specify a line in Space.
7. Specify a plane in Space.
125
4.1 Rectangular Coordinates
The Cartesian plane is formed by two perpendicular number lines, the x-axis and
the y-axis. These axes decide a two-dimensional system of coordinates to classify
points on a plane. You need to add a third dimension to the model to define a
point in space. This three-dimensional model’s geometry is solid analytical
geometry. By passing an axis perpendicular to both the x-axes and y-axes at the
origin,you can create a three-dimensional coordinate system. The three-
dimensional coordination system is divided into eight octants by these planes. The
first octant is the one that is positive for all three coordinates. An ordered triple (x,
y, z) defines a point in space where x, y and z are; x=directed distance from yz-
plane to P y=directed distance from xz-plane to P z=directed distance from xy-
plane to P
126
A three-dimensional coordinate system may have a left or right orientation. As
shown in figure , you should work exclusively with right-handed systems. Octants
II, III and IV are located in a right-handed configuration by rotating in the
counterclockwise direction around the positive axis. Octant V is horizontally
below Octant I. The negative axis then identifies Octants VI, VII, and VIII by
rotating counterclockwise.
Solution
To plot the point (2, -3, -3) notice that x=2, y=-3, and z= 3. To help visualize the
point, locate the point (2, -3) in the plane. The point (2, -3, 3) lies three units
above the cross. The other three points are also shown
127
Figure 4.3: Rectangular coordinates Octants.
128
4.2.1 Distance Formula in Space
The distance between two points in space can be calculated using the distance
formula
d= (x 2 - x1 ) 2 + (y 2 - y1 ) 2 + (z 2 - z1 ) 2 (4.2)
Activity
Finding the Distance Between Two Points in Space
Find the distance between points (1, 0, 2) and (2, 4, -3).
Solution
We know
Figure 4.5
Activity
Midpoint Formula in Space
( 1,y1,z1) and (x2,y2,z2) given
The midpoint of the line segment joining the points (x
by the Midpoint Formula in Space is
129
Solution
Using the mid point formula
Figure 4.6
Figure 4.7
130
If v represents the vector directed from point P(p1,p2,p3) and Q(q1,q2,q3). We can
get the component form of vector v by subtracting the coordinates of the initial
point from the corresponding coordinates of the terminal point.
Figure 4.8
4.3.1 Vectors in 3D
• Two vectors are equal if and only if their corresponding components are
equal.
• The magnitude of w is
w
• the sum of v = hv1,v2,v3i and w = hw1,w2,w3i is
v + w = hv1 + w1,v2 + w2,v3 + w3i
• The scalar multiple of the real number a with vector w is
aw = haw1,aw2,aw3i
• the dot product of v = hv1,v2,v3i and w = hw1,w2,w3i is
v · w = v1w1 + v2w2 + v3w3
131
4.3.2 Angle Between Two Vectors
Figure 4.9
Activity
Finding the Angle Between Two Vectors
Find the angle between u = h1,0,2i and v = h3,1,0i as shown in Figure.
Solution
As we know u·v
cosθ =
as we learned in the previous section the magnitude of a vector w is |w|
| |=
, so
and the dot product of two vectors is defined in the following way v · w =
v1w1 + v2w2 + v3w3
u·v=3
hence we have
132
4.3.3 Parallel Vectors
Scalar multiplication of a positive scalar vector multiples have the same direction
as v, while negative multiples have the opposite direction of v. In general, two
nonzero vectors u and v, are parallel when there is some scalar a such that u = av.
Figure 4.10
Activity
Using Vectors to Determine Collinear Points
Determine whether the points P(2,-1,4), Q(5,4,6)and R(-4,-11,0) are collinear.
Solution
→ →
The component forms of PQ and PR are
→
PQ = 5 −2,4 −(−1),6 −4 = 3,5,2
→
PR = −4 −2,−11 −(−1),0 −4 = −6,−10,−4
→ →
As we can see that PR = −2PQ from which we can conclude that they are parallel
to each other. Hence points P, Q and R lie on the same line.
133
Figure 4.11
Figure 4.12
134
We can say that cosα,, cosα
cos and cosα are the direction cosines of the vector a. The
direction cosines of a nonzero vector a are simply the components of the unit vector
Solution
We know from previous
135
Geometric Properties of the Cross Product
The first property mentioned in the previous section shows that it is not
commutative for the cross product. The property shows in general that the vectors
have the same lengths in opposite directions. Let a and b are two vectors and there
is angle θ between them
a × b is orthogonal to both a and b.
a × b =|a||b|sinθ
• a × b = 0 if and only if a and b are scalar multiples of each other.
• |a × b| = area of a parallelogram having a and b as the adjacent sides.
a × b and b × a are perpendicular to the plane in which a and b are present. One
can remember the direction by comparing the a, b and a × b with unit vectors i,j
and k = i × j respectively, as represented in the figure.
Figure 4.13
Activity
Find a unit vector that is orthogonal to both u =3i−4j + k and v = −3i+6j ?
Figure 4.14
136
Solution
The cross product of u and v will give us the required vector, which will be
orthogonal to both u and v.
We have
know the unit vector which will be orthogonal to both u and v will be defined as
Activity
Display that the quadrilateral is a parallelogram with the following vertices. Then
calculate the parallelogram’s area?
Figure 4.15
Solution
The sides of the quadrilateral corresponding the following four vectors.
137
Because = −AB and = −AD you can conclude that is parallel to and
is parallel to . It follows that the quadrilateral is a parallelogram with and as
adjacent sides. Moreover, because
Let a b and c are three vectors in space, the scalar triple product is defined as
when the vectors a, b and c do not lie in the same plane than the scalar triple
product gives the volume of the parallelepiped.
Figure 4.16
4.4.2 Lines in Space
In a plane, slope is used to identify the equation of a line. In space, the more
appropriate way is to use vectors to find the equation of a line. In the figure,
138
consider the line L through the point P (x1,y1,z1) and parallel to the vector v
(a,b,c).
Figure 4.17
The vector v represents the direction vector for line L and a, b and c are the
direction numbers. One way of describing the line is to say that it consists of all
points Q(x,y,z)) for which the vector PQ is parallel to v.. This means that PQ is a
scalar multiple v of and you can write
PQ = tv
where t defines a scalar number
PQ = x −x1,y −y1,z −z1
PQ = at,bt,ct = tv
Parametric Equations
A line L which is parallel to a vector v = a,b,c and passing through the point
P (x1,y1,z1) is represented b the parametric equations
X = x1 + at
Y = y1 + bt
Z = z1 + ct
when the scalar numbers a, b, and c are non-zero,
non zero, you can eliminate the parameter
t to obtain the symmetric equations of a line
Activity
Find parametric and symmetric equations of the line L that passes through the
point (1,-2,4)
2,4) and is parallel to vector v = (2,4,−4).
139
Figure 4.18
Solution
x1 = 1 y1 = −2 z1 = 4
and direction numbers are
a=2 b=4 c = −4
Now using parametric equations of a line we have
x = 1 + 2t
y = −2 + 4t
z = 4 −4t
Now the symmetric equations can be written as
The parametric equations and symmetric equations are not unique for a given line.
For Example by taking t = 1 in the parametric equations you
or example, in previous Example,
would obtain the point (3,,2,0).
0). With this point the direction numbers will be a = 2, b
= 4 and c = −4 which produces the parametric equations of the following form
x = 3 + 2t
y = 2 + 4t
z = −4t
4.4.3 Plane in space
You’ve seen how to obtain an equation of a line in space and a vector
perpendicular to it from a point on the line. You can now see that it is possible to
obtain an equation of a plane in space from a point in the plane and a normal
(perpendicular) vector to the plane.
140
Figure 4.19
Activity
Find the general form of the equation of the plane passing through the point
(1,3,6) and perpendicular to the vector n = 2i −j + 6k.
Solution
Using the direction numbers for n and the initial point (x1,y1,z1) = (1,3,6)you can
determine an equation of the plane to be
a(x −x1) + b(y −y1) + c(z −z1) = 0
−3(x −1) + 2(y −3) + 1(z −6) = 0
−3x + 2y + z −9 = 0
141
Activity
Find the equation of a plane passing through the points (2,1,1), (0,4,1) and
(−2,1,4).
Figure 4.20
Solution
You need a point in the plane and a vector that is normal to the plane to find the
plane’s formula. The point has three options, but there is no perpendicular vector.
Now taking the cross product of vectors u and v from point the (2,1,1) to the
points (0,4,1) and (−2,1,4) respectively.
The above vector represents perpendicular vector to the given plane. Using the
direction numbers for n and the initial point (x1,y1,z1) = (2,1,1) you can find out
the equation of plane through the standard form of plane represented as
a(x −x1) + b(y −y1) + c(z −z1) = 0
9(x −2) + 6(y −1) + 12(z −1) = 0
9x + 6y + 12z −36 = 0
3x + 2y + 4z −12 = 0
142
By putting each point in the give example we can verify that these points will
satisfy the above equation of plane.
(2
(2,1,1) 3x + 2y + 4z −12= 0
3(2) + 2(1) + 4(1) −12= 0
(0
(0,4,1) 3x + 2y + 4z −12= 0
(−2
−2,1,4) 3x + 2y + 4z −12= 0
Figure 4.21
when the planes are perpendicular to each other n1 ·n2 will be zero and when they
are parallel n1 is a scalar multiple of n2.
Activity
Find the angle between the two planes having the following equations
x −2y + z = 0
2x + 3y −2z = 0
143
and also determine the parametric equation of he intersection line of the planes
Figure 4.22
Solution
The normal vectors perpendicular to the planes are
n1 = 1,−2,1
n2 = 2,3,−2
Now to find out the angle between the two planes we have the following formula
n1 · n2 = h1,−2,1i · 2,3,−2
= 2 −6 −2 = −6
Now multiplying first equation with two and subtracting from second
equation of planes we have
144
2(x −2y + z) = 0
−(2x + 3y −2z) = 0
we have
Simplifying, we get
This tell us the cross product of the normal vectors of the planes is parallel to the
line o intersection of the planes.
When a plane intersects one of the coordinate planes in space, the line of
intersection in the coordinate plane is called the trace of the plane. To draw up a
plane in space, it is important to locate the intersection points in the coordinate
planes with the coordinate axes and their traces.
3x + 2y + 4z = 12
we can find the xy −trace
trace by setting z = 0 and sketching the line
3x + 2y = 12
in the xy plane.
145
The line intersects x −axis at (4,0,0) and y −axis at (0,6,0).
Figure 4.23
Similarly the yz −trace is found by setting x = 0
3x + 4z = 12
The line intersects x −axis at (4,0,0) and y −axis at (0,0,3).
Figure 4.24
Similarly the xz −trace is found by setting y = 0
3x + 4z = 12
The line intersects xy −axis at (4,0,0) and y −axis at (0,0,3).
Figure 4.25
146
In case when the equation of a plane has a variable missing, such as
2x + z = 1
then the plane must be parallel to the axis represented by the missing variable.
Figure 4.26
In case if two variables are missing from the equation of a plane then it is parallel
to the coordinate plane represented by the missing variables.
Figure 4.27
147
Figure 4.28
The distance between a plane and a point is
Self assessment
1. Find the vector v from the following set of figures.
148
2. Find the vector form the following initial and terminal point
i)(−6,4,−2) (1,−1,3)
3) ii)(−1,2,−4) (1,4,−4)
3. Sketch the vector v =h1,1,3i
=h1 with scalar multiples i)2v ii)−v v d)0v.
4. Find the vector u if x = −1,3,2 y = 1,−2,−2 z = 5,00,−5 the
following i)u = x−2 2y ii)2u−4x = z .
5. Find the magnitude of the following vectors a = 7,8,7 b = 1,−2,4
c = 1,3,−1 d = 4,−−3,−7.
6. Find the unit vector in the the direction of vector a and in the direction
opposite to a where a =8i+3j −k.
7. Determine the dot product of vectors a and b where a =4i+4j −k k and b =
2i−5j−8k.
8. What is the angle between two vectors a =0,2,2 and b =3,0,−4?
9. Find out that vector a = −12,6,15 and b =4,−8,−10 are parallel,
orthogonal or neither?
10. Determine that the following points are collinear or not, i)(5,4,1) 1) (7
(7,3,−1)
(4,5,3) ii)(1,3,2) (−1
−1,2,5) (3,4,−1).
11. Find the value of a that satisfy the condition. |av| = 3 where v = i+2
+2j+3k.
12. Find a × b and show that its orthogonal to both a and b where a = 1,−1,0
and b = 0,1,−1.
13. Determine the parallelogram area that has vectors as adjacent sides h4 h4,4,−6i
and 0,4,6.
14. Find that the points P (2,−1,4), Q(3,1,2), R(0,5,6) and S (−1,3,8) 8) are the
vertices of a parallelogram and find its area.
15. Determine the area of triangle having vertices (0,0,0),
(0 (1,2,3), (−3,00,0).
16. Determine the area of triangle hading vertices (2,3,−5),
(2 (−2,−2,0),
0), (3
(3,0,6).
17. Find the triple scalar product of the following vectors.i)
vectors. a = 33,4,4 b =
2,3,0 c = 0,0,6 ii)
ii a = 2i + 3j + k b = i −j c = 4i + 3j + k.
18. Determine the parallelepiped volume of the following figure.
149
19. Determine the parallelepiped volume of the following figure.
(iii)(2,−3,5) x = 5 + 2t,
2 y = 7 −3t, z = −2 + t.
22. Find (a) the set of parametric
parametric equations and (b) if possible, a set of symmetric
equations of the line passing through the points. (i) (2,0,2) (1,4,−3)
23. Draw the line in space, using the following parametric equations
x = 2t,
24. Determine the general form of the equation of the plane passing through the
point (2,1,2)
2) and is perpendicular to vector m = i?
25. Determine the general form of the equation of the plane passing through the
point (5,6,3)
3) and is perpendicular to vector m = −2i +j −2k?
26. Determine the general form of the equation of the plane passing through the
0) and is perpendicular to line x = 3 −t, t = 2 −2t, z = 4 + t?
point (2,0,0)
27. Determine the equation of plane from the following points (0,0,0),(0 0), (1(1,2,3),
(−2,3,3)?
28. Determine the equation of plane from the following pointspo (2,3,−2),
2), (3
(3,4,2),
(1,−1,0)?
29. Determine the general form of the equation of the plane passes through
3) and is parallel to xz plane?
(2,5,3)
30. Determine the general form of the equation of the plane passes through
0) and is parallel to yz plane?
(0,2,4) and (−1,−2,0)
31. Determine the general form of the equation of the plane passes through
(2,2,1) and (−1,1,−1) 2 −3y + z = 3?
1) and is perpendicular to 2x
150
32. Find out that the planes are perpendicular or parallel to each other or none of
them. If they are neither, then find the angle of intersection. 5x−3yy+z = 4, x
+ 4y + 7z = 1.
33. Find out that the planes are perpendicular or parallel to each other or none of
them. If they are neither, then find the angle of intersection. 2x2 −zz = 1, 4x +
y + 8z = 10.
34. Determine parametric equations of the line that passing through (2 (2,3,4) is
parallel to xz and yz plane.. (There are many correct answers.)
35. Determine parametric equations of the line that passing through (2 (2,3,4) is
perpendicular to 3xx + 2y −z = 6. (There are many correct answers.)
36. Determine parametric equations of the line that passing through (5, (5,−3,−4) is
parallel to a = h2,−11,3i. (There are many correct answers.)
37. (a) Determine the angle between the two planes and (b) find parametric
( x +y −2z = 0 ; 2x −y + 3z = 0
equations of their line of intersection. (i)
(ii) 3x −4y + 5z = 6 ; x + y −z = 2 (iii) x + y −z = 0 ; 2x −5y −z = 1
plane.( x+2y+3z = 6 (ii) x+2y = 4
38. Draw the intercepts and sketch the plane.(i)
(iii) 3x + 2y −z = 6
39. Determine the distance between the point and the plane. (i) ( (0,0,0);
0); 88x −4y
+z = 8 (ii) (1,3,4);
4); 4x−5y
4 +2z = 6 (iii) (−2,4,3); 2x+3y +2z = 4.
40. Two lines in space are either parallel or they intersect. Is this statement is
true or false? Justify your answer.
4.6 Summary
1. The distance between two points in space can be calculated using the
distance formula
2. Two
wo vectors are equal if and only if their corresponding components are equal.
3. The magnitude of w is
151
6. The scalar multiple of the real number a with vector w is
aw = aw1,aw2,aw3
7. the dot product of v = v1,v2,v3 and w = w1,w2,w3 is
v · w = v1w1 + v2w2 + v3w3
8. If θ is angle between
en two non-zeros
non vectors u and , then
If two nonzero vectors dot product is zero, then the vector angle is 90o. The
generic unit vectors, for example, are orthogonal to one another.
9. For a non zero vector a
a = a1i + a2j+ a3k
in 3D space the vector a making angle α, β and γ with the x, y,, and z−axes
respectively.
We can say that cos α, cos α and cos α are the direction cosines of the vector a.
Properties of the cross product
10. a × b = −(b × a)
11. a×(b + c) = a × b + a × c
12. c(a × b) = (ca)
a) × b = a × (cb)
(
13. a×0=0×a=0
14. a×a=0
15. a·(b × c) = (a × b)·c
Geometric Properties of the Cross Product
16. The first property mentioned in the previous section shows that it is not
commutative for the cross product. The property shows in general that the
vectors have the same lengths in opposite
oppos directions. Let a and b are two
vectors and there is angle θ between them
17. a × b is orthogonal to both a and b.
18. a × b =|a||b|sinθ
19. a × b = 0 if and only if a and b are scalar multiples of each other.
20. |a × b|| = area of a parallelogram having a and b as the adjacent sides.
Parametric Equations
21. A line L which is parallel to a vector v = a,b,c and passing through the
point P (x1,y1,z1) is represented b the parametric equations
x = x1 + at
y = y1 + bt
z = z1 + ct
when the scalar numbers a, b, and c are non-zero,
non zero, you can eliminate the
parameter t to obtain the symmetric equations of a line
152
22. Consider a plane having a point P(x1,y1,z1) and a perpendicular vector n =
a,b,c as described in the figure. This plane contains all points Q(x,y,z
x,y,z) such
that the vector PQ is perpendicular to n.. Now from dot product we have
n · PQ = 0
a,b,c· PQ = 0
a,b,c· x −x1,y −y1,z −z1 = 0
a −x1) + b(y −y1) + c(z −z1) = 0
a(x
153
Self Assessment Solution
1. (0,6,0) (−2,3,1)
1)
2. (i) (7,−5,5) (ii) (2,2,0)
3.
7. −4
8. About 124.45o
9. Parallel
12. (1,1,1)
13. 56
14. = (1,2,−2)
2) is parallel to = (1,2,−2) = (−3,4,4) is parallel to =
(−3,4,4)
154
18. 2
19. 12
20. 84
23.
24. x −2 = 0
25. −2x + y −2z + 10 = 0
26. −x −2y + z + 2 = 0
27. −3x −9y + 7z = 0
28. 6x −2y −z −8 = 0
29. y −5 = 0 30. y −z + 2 = 0
30. y–5=0
31. 7x + y −11z −5 = 0
32. Perpendicular
33. Perpendicular
34. x=2 y=3 z=4+t
35. x = 2 + 3t y = 3 + 2t z = 4 −t
36. x = 5 + 2t y = −3 −t z = −4 + 3t
155
37. (i) (a)56.9o (b) x = t, y = −7t, z = −3t (ii) (a) 60.7o
o
(b) x = −t + 2, y = 8t, z = 7t (iii) (a)77.8 z = 7t + 1 (b) x = 6t + 1, y = t,
38.
40. False. Lines that do not intersect and are not in the same plane may not be
parallel
Further Readings
1. Precalculus with Limits, Third Edition, Ron Larson and David C. Falvo.
156
Unit – 5
ANALYTICAL GEOMETRY OF
THREEDIMENSION-II
157
CONTENTS
Introduction .................................................................................................159
Objectives ............................................................................................................ 159
5.1 Cylindrical Coordinates ..................................................................................... 160
5.2 Spherical Coordinates ........................................................................................ 161
5.3 Surfaces ................................................................................................................ 164
5.3.1 Cylindrical Surfaces .............................................................................. 164
5.3.2 Quadric Surfaces.................................................................................... 168
5.5.3 Surfaces of Revolution ......................................................................... 174
5.4 Spherical Trigonometry ..................................................................................... 177
5.4.1 Great circle ............................................................................................. 178
5.4.2 Lune ......................................................................................................... 179
5.4.3 Spherical Triangle ................................................................................. 180
5.5 Summary .....................................................................................................181
Further Readings .........................................................................................186
158
INTRODUCTION
OBJECTIVES
After studying this unit you will be able to:
159
5.1 Cylindrical Coordinates
A cylindrical coordinate system is a three-dimensional
dimensional coordinate system defining
point locations by the distance from the selected reference axis, the direction from the
axis relative to the selected reference line, and the distance from the selected
reference plane perpendicular to the axis.
axis. The latter distance is given as a positive or
negative number depending on which side of the point is facing the reference plane.
The distance from the axis can be called the radial distance, while the angular
coordinate is called the angular direction or the azimuth. The radius and the
azimuth are called the polar coordinates, since they correspond to a two
dimensional polar coordinate system in the plane, perpendicular to the reference
line, through the point. The third coordinate may be called the altitude
altitude or height
(if the reference plane is assumed to be horizontal), longitudinal, or axial.
Using the following conversion terms we can get point P coordinates from
cylindrical (r,θ,z)) to cartesian (x,y,z)
( coordinates
x = r cosθ
y = r sinθ
z = z
Similarly the following conversion terms can be used to get point P coordinates
from cartesian (x,y,z)) to cylindrical (r,θ,z)
( coordinates
160
Figure 5.1: Cylindrical coordinates.
The point (0, 0, 0) is called the pole. Moreover, because the representation of a
point in the polar coordinate system is not unique, it follows that the
representation in the cylindrical coordinate system is also not unique.
161
we will us the (ρ,θ,φ)) for the spherical coordinate system.
In a sphericalal coordinate a point P in space is represented by an ordered triple
(ρ,θ,φ)
1. ρ represent the distance of P from the origin(ρ ≥ 0).
2. θ is the same angle use in cylindrical coordinates
3. φ is the angle between the positive z-axis and the line segment OP(0 ≤ φ ≤ π).
Using the following conversion terms we can get point P coordinates from
spherical (ρ,θ,φ)) to cartesian (x,y,z)
( coordinates
x = ρsinφcosθ
y = ρsinφsinθ
z = ρcosφ
Similarly the following conversion terms can be used to get point P coordinates
from cartesian (x,y,z)) to spherical (ρ,θ,φ)
( coordinates
!
To convert coordinates from spherical to cylindrical we have
r2 = ρ2 sin2 φ
θ = θ
z = ρcosφ
To convert coordinates from cylindrical to spherical we have
The spherical coordinate system is useful primarily for surfaces in space that have
a point or center of symmetry.
symmetry
162
Transform the point from rectangular coordinates to cylindrical coordinates.
4. (0,5,1)
5. (2, –2, –4)
6. (1, √3, 4)
13. r2 + z 2 = 5
14. r = 4sinθ
163
Find an equation in rectangular coordinates for the surface represented by the
spherical equation, and sketch its graph.
5.3 Surfaces
In this section we will discus about different surfaces that exist in space or three
dimension. We will discuss cylindrical, quadratic and surfaces of revolution.
164
It’s normally a xyz-space surface. The simplest surfaces are planes, a plane’s
equation is a linear equation which can be written in the form
ax + by + cz+ d = 0
Cylinders in order of complexity are the next surfaces after planes. We consider a
plane curve C and a line L not parallel to C’s plane to understand what these
surfaces are. We mean by a cylinder the geometric figure in space that is
generated (or swept out) by a straight line that moves parallel to L and passes
through C. The cylinder generator is called the moving line. It can be assumed
that the cylinder consists of infinitely many parallel lines called rulings,
corresponding to different generator positions.
165
Figure 5.4:
Activity
Sketch the cylinder
Solution
This seems to be an ellipse equation in the xy-plane.
xy plane. Nevertheless, it is claimed
that this is a cylinder and the rulings of this cylinder are parallel to the z-axis
axis since
the parameter z is absent from the equation. Next draws the ellipse in the xy xy-
plane, then two vertical rulings, then a horizontal cross section above the xy xy-
plane. Given our figure’s limitations (which we hopehope students can try to overcome
through an active use of imagination), it should be noted that in both directions all
rulings on a cylinder stretch to infinity. The elliptic ring is called this elliptical
cylinder.
166
It is clear that for a cylinder with rulings parallel to any coordinate axis, this
discussion can be conducted. Therefore, we have the conclusion that any equation
with one variable missing in rectangular coordinates x, y, z is a cylinder whose
rulings are parallel to the missing variable’s axis.
Activity
Sketch the cylinder z = x2.
Solution
In the xz-plane, this is the equation of a parabola with a vertex at the origin that
opens in the positive z-direction. We realize, however, that we are dealing with a
cylinder, and since the parameter y is absent from the equation, this cylinder’s
rulings are along the y-axis. The parabola in the xz-plane is first drawn, then
several rulings, then a second parabolic cross section on the right side of the xz-
plane. It is possible to describe this surface as a parabolic cylinder.
Another way to generate a surface is to rotate the curve (in space) about a line L
in your plane using a plane curve C. The resulting surface is called a surface of
revolution with axis L. Through measuring their areas as an application of definite
integrals, we became acquainted with surfaces of revolution. The equations of
these surfaces are now being considered.
Suppose, for example, that the curve C lies in the yz-plane and has equation
f (y,z) = 0 (5.3)
As this curve moves around the z-axis, the resulting surface has a standard
point P = (x, y, z) from a point Q on C. Since Q is on C, it satisfies (5.3) its
coordinates (yo, zo),
f (y0,z0) = 0 (5.4)
167
But the relation of P to Q tells us that zo = z and y0 = x 2 y 2 , so (5.4) yields
=0 (5.5)
as the equation of the surface of revolution. In short, as Q swings out to the point
P on the surface, the distances QR and PR to the z-axis axis are equal, and we get
2 2
equation (5.5) by replacing y in (5.3) by x y . Equation (5.5) assumes C to
be y ≥ 0. If y is positive in some parts of C and negative in others, we need to
replace y in (5.3) with ± x 2 y 2 in order to get
Like the formula of the entire surface. Typically, squaring will eradicate the
awkward radical with its ± symbol.
We assume the coefficients A, B,.... is not all is zero, so the equation degree is
actually 2 instead of 1 or 0. This equation’s graph is called a quadric surface.
Faces like spheres and parabola, elliptic and hyperbolic cylinders, but there are
also a number of others. Never the less, if we set aside the common case of
cylinders, then by correct rotations and translations of the coordinate axes
axes-which
we are not addressing-any
any form equation (5.6) can be simplified.
The intersection of a surface with a plane is called the trace of the surface in the
plane. Determining the traces in some well-chosen
well chosen planes is helpful in visualizing
a surface in space. The quadric surface traces are conics. Such traces and the
typical formula form of each quadric surface.
168
The ellipsoid
TracePlane
EllipseParallel to xy-plane
plane
EllipseParallel to xz-plane
plane
EllipseParallel to yz-plane
plane
The surface is a sphere when a 6= b 6= c 6= 0.
Trace Plane
Ellipse Parallel to xy-plane
xy
Hyperbola Parallel to xz-plane
xz
Hyperbola Parallel to yz-plane
yz
169
The axis of the hyperboloid corresponds to the variable whose coefficient is
negative.
The hyperboloid of two sheets
170
Trace Plane
Ellipse Parallel to xy-plane
xy
Hyperbola Parallel to xz-plane
xz
Hyperbola Parallel to yz-plane
yz
The axis of the cone corresponds to the variable whose coefficient is negative.
The traces in the coordinate planes parallel to this axis are intersecting lines.
Trace Plane
Ellipse Parallel to xy-plane
xy
Parabola Parallel to xz-plane
xz
Parabola Parallel to yz-plane
yz
The axis of the paraboloid corresponds to the variable raised to the first power.
171
Trace Plane
Hyperbola Parallel to xy-plane
xy
Parabola Parallel to xz-plane
xz
Parabola Parallel to yz-plane
yz
The axis of the paraboloid corresponds to the variable raised to the first power.
Activity
4 2 −3y2 + 12z2 + 12 = 0.
Classify and sketch the surface 4x
Rearranging we have
From the previous section we can conclude that the surface is a two two-sheet
hyperboloid with the y-axis
axis as its
its axis. It helps in finding traces in the coordinate
planes to sketch the graph of this surface.
We have
Figure 5.13:
172
Activity
Classify and sketch the surface x + y2 + 4z2 = 0.
Solution
Since x is only raised to the first power, the surface is a paraboloid.
The x-axis
axis is the axis of the paraboloid. The equation is in standard form
For a quadric surface not centered at the origin, you can form the standard
equation by completing the square.
Figure 5.14:
173
Next, complete the square for each variable and write the equation in standard
form.
From this equation, you can see that the quadric surface is an ellipsoid that is
centered at (2,−1,1).
Figure 5.15:
The special type of surface you will study is a surface of revolution. We will look
at a procedure for finding its equation. Consider the graph of the radius function
y = r (z)
In the plane of the yz,, when the graph revolves around the z-axis, axis, it forms a
revolutionary surface. The plane z = z0 trace of the surface is a circle whose radius
is r(z0) and whose equation is
x2 + y2 = [r (z0)]2
Replacing z0 with z generates a formula valid for all z values. Likewise, formulas
for revolution surfaces for the other two axes can be obtained, and the resu
results are
summarized as follows.
If the graph of a radius function r is rotated around one of the coordinate axes,
then one of the forms described below is the formula of the resulting surface of
revolution.
174
Figure 5.16:
Activity
Find an equation for the surface of revolution formed by revolving (a) the graph
of y = 1/z about the z-axis 9 2 = y3 about the y-axis.
axis and (b) the graph of 9x
Solution
a. An equation for the surface of revolution formed by revolving the graph of
b. To find an equation for the surface formed by revolving the graph of 9x2 =y3
about the y-axis,
axis, solve for x in terms of y to obtain
The generating curve for a surface of revolution is not unique. For instance, the
surface
x2 + z2 = e−2y
can be formed by revolving either the graph of
x = e−y
175
about the y-axis
axis or the graph of
z = e−y
about the y-axis.
Figure 5.17:
Activity
Find a generating curve and the axis of revolution for the surface x2 + 3y2 + z2 = 9.
Solution
The equation has one of the forms listed below.
1. Revolved about the x-axis:
x y2 + z2 = [r (x)]2
2. Revolved about the y-axis:
y x2 + z2 = [r (y)]2
3. Revolved about the z-axis:
z x2 + y2 = [r (z)]2
Because the coefficients of x2 and z2 are equal, you should choose the second form
and write
x2 + z2 = 9 −3y2
The y-axis
axis is the axis of revolution. You can choose a generating curve from
plance trace x2 = 9 −3y2
either of the traces xy−plance
yz−plance trace
z2 = 9 −3y2
For instance, using the first trace, the generating curve is the semiellipse
Figure 5.18:
176
Figure 5.19:
5.4 Spherical Trigonometry
For many centuries, geometry has developed and evolved. Its uses are vast and our
daily lives continue to be affected. In general, the analysis of the sphere has a
unique story of its own and has two major turning points. This research began with
the astronomy drive first and was developed by the Greeks in depth. There is
speculation that mathematical discoveries were made about the sphere as early as
the second century, but there is no proof for that. Menelaus of Alexandra’s
Alexandra’s research
was the major transition to understanding spherical geometry. The equations he
discovered were accepted for a long time, such as measuring a sphere’s
circumference and measuring arc lengths, and no further study was needed.
Religious matters
atters were the next major motivation for studying spherical
trigonometry; Islamic religion demands that Mecca’s path is always understood
for daily prayer. During the time of Islamic Enlightenment, the findings of
Menelaus were further developed. There is some debate about the discovery of
spherical triangles of the Law of Sines. Possible sources of this finding come from
the discussion about the two Muslim scholars, Abu Nasr. Who was ever
responsible for the progress in the Sines Law later allowed a more concise
evidence to be developed; as well as leading to other spherical trigonometry
theorems and identities. Euclid is another big name in geometry. He made a
significant impact later in the third century; while the spherical trigonometry
method does not have parallel lines, Euclidean geometry provided some insight
into spherical behavior. Elements Euclid published equations in his research that
help us lead to the Pythagorean theorem and the Cosines Law While
mathematicians provided insight into this study study area, many influences on
spherical trigonometry also came from the science field. In the late Renaissance
period, further exploration of the behavior of arcs and angles became common.
John Napier, a Scottish physicist living around the 17th century, was the first to
deal with the right spherical triangles and their essential identities. The law of
cosines for spheres has been discovered using Napier’s Rules.
Spherical trigonometry is the branch of spherical geometry that deals with the
relationships between
een trigonometric functions of the sides and angles of the
spherical polygons (especially spherical triangles) defined by a number of
177
intersecting great circles on the sphere. Spherical trigonometry is of great
importance for calculations in astronomy, geodesy and navigation.
A sphere is a solid surrounded by a surface from which that point is equally distant
from a fixed point called the center of the sphere. The straight line that connects
every point with the middle of the surface is called a radius. A diameter is called a
straight line drawn through the middle and finished both ways by the ground.
The section of the surface of a sphere by a plane is called a great circle if the plane
passes through the centre of the sphere, and a small circle if the plane does not
pass through the centre of the sphere. Thus the radius of a great circle is equal to
the radius of the sphere.
A plane can be drawn through the center of a sphere and any two points on the
surface; and only one plane can be drawn, except when the two points are the
ends of a sphere diameter, and then an infinite number of such planes can be
drawn. Thus only one great circle can be drawn on the surface of a sphere through
two given points, except when the points are the limits of a sphere diameter. If
only one great circle can be drawn through two points, the great circle is
unequally divided at the two points; for brevity we shall speak of the shorter arc
of the two arcs as the arc of a great circle joining the two points.
A greatcircle of a sphere, also known as an orthodrome, is the intersection of the
sphere and a plane going through the sphere’s center point. A great circle is the
largest circle on any given sphere that can be drawn. Any diameter of any
greatcircle coincides with a sphere diameter and all large circles therefore have
the same center and circumference as each other. The particular case of a
spherical circle is in contrast to a small circle, that is to say, a sphere intersection
and a plane that does not move through the center. That circle is a great circle
with exactly one sphere in Euclidean 3-space.
178
There is a unique great circle through the two points for most pairs of distinct
points on a sphere’s surface. The exception is a pair of antipodal points
(Antipodal points on a circle are 180 degrees apart.) for which many great circles
exist infinitely. The shortest surface-path between them is the minorarc of a
greatcircle between two points. In this sense, the small arc in Euclidean geometry
is analogous to “straight lines.” The size of a greatcircle’s minorarc is taken as the
distance between two points on the surface of a Riemannian geometry sphere
where such greatcircles are called Riemannian circles. Such greatcircles are the
sphere’s geodesics.
Some examples of great circles on the celestial sphere include the celestial
horizon, the celestial equator, and the ecliptic. Great circles are also used as rather
accurate approximations of geodesics on the Earth’s surface for air or sea
navigation (although it is not a perfect sphere), as well as on spheroidal celestial
bodies. The equator of the idealized earth is a great circle and any meridian and its
opposite meridian form a great circle. Another great circle is the one that divides
the land and water hemispheres. A great circle divides the earth into two
hemispheres and if a great circle passes through a point it must pass through its
antipodal point.
5.4.2 Lune
A lune is a part of the sphere which is captured between two great circles. This
definition is relevant because it started the ability to capture shapes on a sphere.
This definition itself is not extremely significant, but it is through this shape
which we can form other shapes on the sphere.
Figure 5.21: The two great circles are shown as thin black lines, whereas the
spherical lune (shown in green) is outlined in thick black lines.
179
5.6.3 Spherical Triangle
A spherical triangle is the intersection of three distinct lunes.
Spherical Trigonometry investigates the relations which subsist between the
angles of the plane faces which form a solid angle and the angles at which the
plane faces are inclined to each other.
Suppose that the angular point of a solid angle is made the centre of a sphere; then
the planes which h form the solid angle will cut the sphere in arcs of great circles.
Thus a figure will be formed on the surface of the sphere which is called a
spherical triangle if it is bounded by three arcs of great circles; this will be the
case when the solid angle is formed by the meeting of three plane angles. If the
solid angle be formed by the meeting of more than three plane angles, the
corresponding figure on the surface of the sphere is bounded by more than three
arcs of great circles, and is called a spherical
spherica polygon.
The three arcs of great circles which form a spherical triangle are called the sides
of the spherical triangle; the angles formed by the arcs at the points where they
meet are called the angles of the spherical triangle.
Thus, let O be the centre
tre of a sphere, and suppose a solid angle formed at O by the
meeting of three plane angles. Let AB, BC, CA be the arcs of great circles in which
the planes cut the sphere; then ABC is a spherical triangle, and the arcs AB, BC,
CA are its sides. Suppose Ab the tangent at A to the arc AB, and Ac the tangent at
A to the arc AC, the tangents being drawn from A towards B and C respectively;
then the angle bAc is one of the angles of the spherical triangle. Similarly angles
formed in like manner at B and C are the
the other angles of the spherical triangle.
Self Assessment 5.2
Describe and sketch the surface.
1. y2 + z2 = 9
2. 4x2 + y2 = 4
Classify and sketch the quadric surface
180
Find an equation for the surface of revolution formed by revolving the curve in
the indicated coordinate plane about the given axis.
9. z = 5y (Equation of curve) yz−plane(Coordinate Plane) y−
axis(Axis
(Axis of revolution)
10. y3 = 8z (Equation of curve) yz−plane(Coordinate Plane) z−
axis(Axis
(Axis of revolution)
11. xy= = 2(Equation of curve) xy−plane(Coordinate Plane) x−
axis(Axis
(Axis of revolution)
5.7 Summary
181
• The elliptic paraboloid
• The special type of surface you will study is a surface of revolution. We will
look at a procedure for finding its equation. Consider the graph of the radius
function
y = r (z)
182
Solution of Self Assessment 5.2
1.
2.
183
3.
4.
5.
6.
7.
184
8.
9. x2 + z2 = 25y2
10. x2 + z2 = 4z2/3
185
Further Readings
1. Brummelen, Glen Van. Heavenly Mathematics: The Forgotten Art of
Spherical Trigonometry. Princeton: Princeton UP, 2013. Print.
186
Unit – 6
187
CONTENTS
Introduction .................................................................................................189
Objectives ...................................................................................................189
6.1 First Order Differential Equation ................................................................190
6.2 Types of Differential Equations ..................................................................191
6.3 Order and degree of Differential equations ................................................192
6.4 Initial and Boundary Values Conditions .....................................................197
6.5 Separable Variables Differential Equations ................................................201
6.6 Homogeneous Differential equation ...........................................................205
6.7 Integrating Factors .....................................................................................210
Answers ......................................................................................................216
Further Readings .........................................................................................220
188
INTRODUCTION
This unit is an introduction to ordinary differential equations. Differential
equations exists in the mathematical formulation of many problems in science and
engineering i.e. study of chemical reactions, determination of curves which give
geometrical properties, determining the motion of rocket, projectile, planet or
satellite. Similarly we use differential equation to find the charge or current in an
electric circuit.
In this unit we also described the main ideas to solve certain differential
equations, such as first order differential equations. We introduced boundary
value problem and initial value problems. We show particular techniques to solve
the specific types of first order differential equations. These techniques were
developed in the 18th and 19th centuries and the equations include linear equations,
separable equations, Euler homogeneous equations and exact equations.
OBJECTIVES
After studying this unit, prospective teachers will be able to:
1. Describe different types of Differential Equations
2. Find the order and degree of Differential equations
3. Explain initial and Boundary Values Conditions
4. Solve by separating variables of Differential Equations
5. Describe homogeneous Differential equation
6. Find Integrating Factors
189
6.1 First Order Differential Equation
Differential equations exists in the mathematical formulation of many problems in
science and engineering i.e. study of chemical reactions, determination of curves
which give geometrical properties, determining the motion of rocket, projectile,
planet or satellite. Similarly we use differential equation to find the charge or
current in an electric circuit. The word differential and equation clearly suggest
solving a few kind of equation which contains derivatives, ,… .
In the previous study of algebra, we have spent a good amount of time to solve
algebraic equation such that for the unknown x. in this unit our
task will be to solve a differential equation. In the previous study we briefly
examine the relation between differential equations and the real world. We learnt
derivatives or involve rate of change such that we came to know the practical
approach of differential equation such as how fast does the disease spread? How
fast does a population change? But this is not the complete story, about the course
under discussion. As the course unfolds, you will observe that there is more to the
study of differential equations than just learn methods that someone has devised
to solve them.
6.1.2 Definition
The equation involving one dependent variable and its derivatives with respect to
one or more independent variables is said to be differential equation (DE).
Following are the examples of differential equations.
i.
190
ii.
iii.
iv.
v. +
Before proceeding further, we shall classify differential equation by type, order,
degree and linearity.
Remember that ordinary derivatives can be written by using either the Leibniz
y can be written as or .
191
Unknown function or dependent variable
Independent variable
We can quickly observe that the symbol y now represents a dependent variable,
whereas it is the independent variable. You should also be familiar that in
physical science and engineering, Newton’s dot notation is sometimes used to
represent derivatives with respect to time. So the differential equation
becomes . Partial derivatives are usually denoted by a subscript notation
indicating the independent variables.
For example, the equation can be written in subscript notation
as = .
The degree of a differential equation is the degree of the highest derivative that
occurs in the equation.
192
The differential equations mentioned in 6.1 are of the following orders and
degrees.
(i) Order 1, Degree 1
(ii) Order 2, Degree 1
(iii) Order 2, Degree 2
(iv) Order 1, Degree 1
(v) Order 2, Degree 1
In symbols we can show an nth order ODE in one dependent variable by the
general form (1)
So when it suits our purposes, we shall use the normal forms and
to denote the general first and second order equations.
193
Or
Two significant special cases of (2) are linear first order DE for and linear
second order DE for :
_____________ (3)
___________(4)
In the additive combination on the L.H.S of equation (2) we see that the
characteristic two properties of a linear ODE are follow:
The dependent variable y and all its derivatives are of the
first degree, that is, the power of each term involving y is 1.
The coefficients of depend at most on the
independent variable .
The equations:
are linear first, second and third order differential equations respectively. We have
just demonstrated that the first equation is linear in the variable y. It can also be
written in the alternative form such that
We have observed that in a linear ordinary differential equation have the
following qualities
Dependent variable and its derivatives all have degree one
no product exists between the dependent variable y or any of its derivatives
appear
no transcendental function of y and / or its derivatives occur
194
number of arbitrary constants equal to the order of the equation is called general
solution or integral of the differential equation. A solution obtained from the
general solution by giving particular values to the constants is called a particular
solution or integral. The graph of a particular integral is called an integral curve.
Remarks
Equation (2) is called the general solution of the differential equation (1).
Theorem 6.1 demonstrates that equation (1) has infinitely many solutions,
one solution for each value of the constant c, which is not determined by the
equation.
Theorem makes a perfect sense that c is a constant in the solution of
differential equation. The differential equation consists of a first derivative
of the unknown function , so finding a solution of differential equation
need one integration. Every indefinite integration introduces an integration
constant c.
195
be formed by elimination of b from (1). is the general solution of
.
Example 6.2 From the differential equation by eliminating two constants L and
M from the relation
Solution: It is clear that three equations are required to eliminate two unknown
and . We obtain two other needed equations by successive differentiation of
(1). Hence:
__________ (
_________ (
From ( , (2) and (3) we obtain
Example 6.3 Find the differential equation of all parabola whose axis is parallel
to the y-axis is _________ (1)
In order to obtain the differential equation, we have to eliminate and from
(1) and three more equations. Differentiating (1) successively, we have
The last equation does not contain any of the constants and Hence
is the differential equation of all parabolas whose axes are parallel to the
axis y
196
6.4 Initial and Boundary Values Conditions
We have noticed that general solution of a differential equation contains the same
number of arbitrary constants as is the order of the equation. Sometimes we need
to find the solutions of differential equations subject to supplementary conditions.
Two types of conditions will be often encountered.
If the conditions are imposed to one value of the independent variable such that
at [written as and at , then
they are commonly called the initial value conditions or one-point boundary
conditions where is termed as the initial point. An initial value problem
consists of a differential equation (DE) of any order together with a collection of
initial value conditions that must be satisfied by the solution of the differential
equation and derivatives at the initial points.
The names “initial value problem (IVP)” and “boundary value problem (BVP)”
come from physics. The example of the former is to determine Newton’s
equations of motion for the position function of a point particle that starts at a
given initial position and velocity. An example of the latter is to find the
equilibrium temperature of a cylinder bar with thermal insulation on the round
surface and held at constant temperatures at the top and bottom sides.
197
Example 6.4: Solve such that
Solution:
This is an initial value problem.
By integrating
We have
Solution: Here we observed that both the conditions relate to one value of , this
is an initial value problem. So the general solution of the given equation is:
_______ (1)
The initial conditions implies that:
conditions
198
Solution: we have
By differentiating
Again differentiating
solution of (1) is
Solution: We know the general solution of the given differential equation is:
Activity 6.1
1. Find constant a, b, so that is a solution of IVP
199
Exercise 6.1
State order and degree of each equation and determine whether the equation is
linear or non-linear.
(i) 2
(ii)
(iii)
(iv)
(v)
(vi)
(vii)
(viii)
(ix)
(x)
Q2. From the differential equation (DE) of which the given function is a
solution:
(i)
(ii) , being an arbitrary solution.
(iii)
(iv)
(v)
200
general solution.
(ii) , given that the DE has
as the general solution.
(iii) , , , where
is the general solution of the given DE.
Q5. Solve the initial value problem , given that the
201
By integrating we have,
Or, which is the required solution.
Example 6.9: Solve
;
is the required solution.
On integrating, we obtain
is the required solution.
On integrating, we obtain:
Or,
Now,
202
Similarly,
, where c
Thus , , it follows that:
This is the required solution.
Now,
Therefore,
Putting and we get
and ,Thus equation (1) becomes
Integrating, we have:
Or, =
= __________ (2)
Since is positive.
When and so from (2) we get:
or,
Thus (2) yields
is the required solution.
Activity 6.2
203
2.
3.
4.
5.
Exercise 6.2
2.
3.
4.
5.
6.
7.
8.
9.
10.
11.
12.
13. Solve the initial value problem. ,
14. Solve the IVP: ,
15. Solve the IVP: ,
204
6.6 Homogeneous Differential Equation
Sometimes a differential equation is not a separable but we can transform it into a
separable equation changing it to the unknown function.
Then we will solve the differential equation by the variables separation method.
Finally to obtain the general solution of the required differential equation we will
replace by .
205
Here the equation is homogeneous. Setting and in (1),
we have
Therefore,
Or,
Or,
Or,
This is the required solution.
Solution: Here
206
Integrating, we get
Or
i.e.,
Applying the given conditions
__ (2)
So the above equation becomes
207
Then (2) reduces to the homogeneous equation:
_____ (3)
Equation (3) is homogeneous in the variable X and Y.
2. When , then substitute and the given equation
is ,
Now, equation (2) reduces to which is homogeneous.
Now substitute in the above homogeneous equation we have:
Or
____ (3)
Equation (3)is a separable. Therefore,
By integrating, we have
208
where
Or
Now replacing by , it becomes
Or
But hence equation (4) takes the form
, This is the required solution.
Since , we put
Put
and or then equation (1) becomes
_______ (3)
Integrating, we have
Replacing by , we obtain
209
Activity 6.3
1. Solve:
2. Solve:
3. Solve:
Exercise 6.3
Solve (problems 1-5):
1.
2.
3.
4.
5.
Solve:
8.
9.
10.
210
and , if expression (1) is an exact
differential then
=0 ________ (2)
Equation (2) is called exact differential equation.
The number of integrating factors (I.F) of the differential equation can be infinite.
We will mention some rules (without proofs) to find integrating factors of
equations of special types.
If the differential equation (DE)
____ (1)
isn’t exact and
Such that P is a function of only then equation (1) has an I.F which
also dependson , is solution of i.e.,
Remember that,
211
If , and is a function of only, so the differential equation
has an I.F
In case the equation is homogeneous equation and
then is an I.F of the given homogeneous equation.
(ii)
(iii)
(iv)
(v)
(vi)
To understand this concept you must follow the example given below:
Integrating, we have
212
is the general solution.
Example 6.18: Solve
Solution: Here
Integrating factor
Multiplying the given equation by the integrating factor, we have
, or
Integrating, we have
Integrating factor
Now by integrating
213
Example 6.20: Solve
Solution: Here
So , , and
Integrating factor
We have
, or
Example 6.21Solve
Solution:
So integrating factor
Integrating, we have
Activity 6.4
Solve (Problem 1-5)
1.
214
2.
3.
4.
5.
Exercise 6.4
Solve (problems 1-5 by integrating factor)
1.
2.
3.
4.
5.
215
ANSWERS
Activity 6.1
1.
2.
3.
4. + with
5.
Exercise 6.1
Q1. i 1st order, 1st degree, Non-linear ii 3rd order, 1st degree, Linear
iii 2nd order, 3rd degree, Non-linear iv 2nd order, 1st degree, Linear
v 2nd order, 1st degree, Non-linear vi 2nd order, 1st degree, Linear
vii 3rd order, 4th degree, Linear viii 4th order, 1st degree, Linear
ix 2nd order, 3rd degree, Non-Linear x 3rd order, 1st degree, Linear
Q2. i
ii
iii
iv
Q3. i
216
ii
Q4. i
ii
iii
Q5.
Q6.
Q7.
Q8.
Activity 6.2
1.
2.
3.
4.
5.
Exercise 6.2
1.
2.
217
3.
4.
5.
6. if and
if and
7.
8.
9.
10.
11.
12.
13.
14.
15.
16.
Activity 6.3
1.
2.
3.
Exercise 6.3
1.
2.
218
3.
4.
5.
6.
7.
8.
9.
10.
Activity 6.4
1.
2.
3. +12
4.
5.
Exercise 6.4
1.
2.
+
3.
4.
5.
219
Further Readings
2. Thomous’s Caculus
220
Unit – 7
221
CONTENTS
Objectives ...................................................................................................223
Introduction ................................................................................................223
7.1 Differential Equations .................................................................................224
7.2 Linear Differential Equations .....................................................................224
7.2.1 Homogeneous DE ..........................................................................225
7.2.2 Non-Homogenous DE ....................................................................225
7.2.3 Steps to follow for Solving a Linear First-Order Equation.............226
7.3 Exact Differential Equation ........................................................................234
7.3.1 Theorem Criterion for an Exact Differential ..................................234
7.3.2 Solution of Exact differential equation ...........................................235
7.3.3 Integrating Factors .........................................................................239
7.4 Bernoulli Equation ......................................................................................241
7.4.1 Transformation to a linear differential equation .............................241
7.5 Lagrange and Clairaut Equations ...............................................................242
Further Readings .........................................................................................246
222
INTRODUCTION
In this unit we will learn the concept of differential educations and their types
which are linear, homogeneous and non homogenous differential educations.
Exact differential educations, Bernoulli Equations, Lagrange and Clairaut
Equations are also discussed in detail this unit will also explain the step by step
solution of these kinds of differential equations. A detailed overview of the
theorems Criterion for an exact differential and its use, methods of integrating
factors and transformation to linear differential equations has also been given.
Unit is enriched with the examples and activities for further concept clarity.
OBJECTIVES
After studying this unit, prospective teachers will be able to:
1. Solve the linear differential equations.
2. Solve homogeneous the linear differential equations.
3. Solve non-homogeneous the linear differential equations.
4. Explain and solve exact differential equations.
5. Describe the method to find integrating factors for solution of differential
equations.
6. Solve Lagrange and Clairaut Equations.
223
7.1 Differential Equation
A differential equation is a mathematical equation that relates with its derivatives
some variable. For applications, functions generally represent physical quantities,
derivatives represent their rate of change, and a relationship between the two is
defined by the differential equation. Because such relationships
relationships are extremely
common, differential equations in many disciplines, including engineering,
physics, economics, and biology, play a prominent role.
Differential equations are studied in pure mathematics from several different
perspectives, often concerned
concerned with their solutions the set of functions that satisfy
the formula. Only the simplest differential equations can be solved by direct
formulas; however, it is possible to determine other properties of solutions of a
given differential equation without knowing
kno their exact form.
If there is no closed-form
form expression for the solution, computers can approximate
the solution numerically. Dynamic systems theory emphasizes the qualitative
analysis of systems described by differential equations, while numerical m methods
have been developed to determine solutions with a degree of accuracy.
7.2 Linear Differential Equations
A first-order
order differential equation of the form
is said to be a linear equation in the dependent variable y.
We seek a solution on an interval I for which both functions P (x)) and f (x) are
continuous.
In the discussion that follows, we illustrate a property and a procedure and end up
with a formula representing the form that every solution of (2) must have. But
more than the formula, the property and the procedure are important, because
these two concepts carry over to linear equations of higher order.
The differential equation (2) has the property that its solution is the sum of the two
solutions, y = yc+ yp, where ycis a solution of the associated homogeneous equation
224
7.2.1 Homogeneous DE
The homogeneous equation 7.3 is also separable. This fact enables us to find ycby
writing 7.3 as
Now fact is
we have
Hence
225
and
y = yc+ yp
(7.11). The left side of (7.11) is recognized as the derivative of the product of e
P(x) dx and y. This gets us to (7.10). We then integrate both sides of (7.10) to get
the solution (7.9). Because we can solve (7.2) by integration after multiplication
by eʃ p(x)dx, we call this function an integrating factor for the differential equation.
For convenience we summarize these results. We again emphasize that you
should not memorize formula (7.8) but work through the following two two-step
procedure each time.
7.2.3 Steps to follow for Solving a Linear First-Order
First Equation
• Put a linear equation of form
form (7.1) into standard form (7.2) and then
determine P(x)) and the integrating factor eʃp(x)dx.
• Multiply (7.2) by the integrating factor. The left side of the resulting
equation is automatically the derivative of the integrating factor and y. Write
226
The integrating factor is eʃp(x)dx = eR (−4)dx= e−4x.
Now multiply the integrating factor with the given equation we have
we have
Simplifying
where as
f (x) = x5ex
The p and f are continuous on the interval (0,∞).
(0 ∞). Now multiplying the integrating
factor with the given equation we have
simplifying
227
It follows from integration by parts
we have
x−4y = xex−ex + c
The general solution on interval (0,∞)
(0 is
y = x4ex (x −1) + cx4
y = x5ex −x4ex + cx4
Example 7.3
Find the general solution of differential equation
Now multiplying the integrating factor with the given equation we have
we have
Integrating we have
Notice in the preceding example that x = 3 and x = −3 are singular points of the
equation and that every function in the general solution y = c/ x 2 9 is
discontinuous at these points. On the other hand, x = 0 is a singular point of the
differential equation in Example 2, but the general solution y = x5ex −x4ex + cx4 is
one parameter family is continuous at x =
noteworthy in that every function in this one-parameter
0 and is defined on the interval (−∞,∞)
( and not just on ∞ as stated in the solution.
However, the family y = x5ex −x4ex + cx4 defined on (−∞,∞) ∞) cannot be considered
the general solution of the DE, since the singular point x = 0 still causes a problem.
Example 7.4
Solve the initial value dy/dx + y = x; y (0) = 4.
Solution:
The equation is in standard form, and P(x) = 1 and f(x) = x are continuous on the
∞). The integrating factor is eʃP(x)dx = eʃdx = ex, and so integrating
interval (−∞,∞).
228
Now integrating we have
Simplifying we have
yex= xex−ex + c
The general solution has the following form
y = x −1 + ce−x
As we know from the initial condition that at x = 0 we have y = 4, so putting the
respective values in the general solution we have
4 = 0 −1 + ce−0
Simplifying we have
c=5
Hence the solution of the problem on the interval (−∞,∞) is
y = x −1 + 5e−x
The general solution of every linear first-order
first order differential equation is a sum of
two special solutions: yc, the general solution of the associated homogeneous
equation (7.3), and yp, a particular solution of the nonhomogeneous equation (7.2).
In Example 7.4 we identify
yc= ce−x
and
yp= x −1
229
values of x. We say that yc= ce−x is a transient term since yc→ 0 as x → ∞. While
this behavior is not a characteristic of all general solutions of linear equations, the
notion of a transient is often important in applied problems.
Example 7.4
Solve the following differential equation
Solution:
The graph of the discontinuous function f is shown in the figure.
Now solving the differential equation y(x) first on the interval [0,1]
1] and then on
∞). For 0 ≤ x ≤ 1 we have
the interval (1,∞).
The integrating factor is eʃP(x)dx = eʃdx = ex. Multiplying the integrating factor with
the given equation we have the following form
230
Multiplying the integrating factor with the given equation we have the following form
Integrating we have
exy= c2
further simplifying
y = c2e−x
In the end we can write
Example 7.5
Solve the differential equation with initial-value
initial
Solution:
The differential equation is already in standard form, and so we see that the
integrating factor is
ʃ R
e P(x)dx = e (−2x)dx= e−x2
Now multiplying the given equation with the integrating factor we have
231
Simplifying we have
Simplifying we have
Rearranging we have
The graph of solution is shown in figure. it was obtained using computer base
software MATHEMATICA.
232
Self Assessment 7.1
Solve the following differential equations
233
17. Express the solution of the initial value problem dy/dx −2xy = 1; y (1) = 1 in
terms of error function.
18. The sine integral function is defined as , where the
integrand is defined to be 1 at x=0. Express the solution of the initial value
problem x3dy/dx −2x2x2y = 10sinx; y (1) = 0 in terms of Si(x).
(7.12)
Now if f (x,y) = c, it follows from
To our purposes, turning the problem around is more important; in other words,
given a first order differential equation such as (7.14), we should understand that
it is equal to the differential d(x2 −7xy + y3) = 0?
Let M(x, y) and N(x, y) be continuous and have continuous first partial derivatives
in a rectangular region R defined by a < x <b,c< y < d.. Then a necessary and
sufficient condition that M(x,y)dx
M + N(x,y)d be an exact differential is
234
PROOF:: For simplicity let us assume that M(x,y) and N(x,y)) have continuous first
partial derivatives for all (x,y).
( Now if the expression M(x,y)dx + N(x,y)dy
dy is exact,
there exists some function f such that for all x in R,
therefore
and
The equality of the mixed partial is a consequence of the continuity of the first
partial derivatives of M(x,y
x,y) and N(x,y).
We can find f by integrating M(x,y) with respect to x,, while holding y constant:
this gives
(7.17)
Finally, integrate (7.17) with respect to y and substitute the result in (7.16).
The implicit solution of the equation is f(x,y) = c.
Some observations are in order. First, it is important to realize that the expression
N(x,y) −(∂/∂y)ʃ M(x,y)dx
dx in (7.17) is independent of x, because
235
Second, we could just as well start the foregoing procedure with the assumption
that ∂f/∂y = N(x,y).
). After integrating N with respect to y and then differentiating
that result, we would find the analogues of (7.16) and (7.17) to be, respectively
f (x,y) = ʃ N (x,y)dy+ h(x)
If you find that integration of ∂f/∂x = M(x,y) with respect to x is difficult, then try
integrating ∂f/∂y = N(x,y)) with respect to y.. In either case none of these formulas
should be memorized.
Example7.6
Solve the following exact differential equation
Solution:
In above equation we have
M (x,y)
( = 2xy N (x,y) = x2 −1
we have the
Thus the equation is exact and so there exist a function f (x,y) such that
and
It follows that
1 and g (y) = −y
Hence, f (x,y) = x2y −y,, and so the solution of the equation in implicit from is x2y
−y = c.. The explicit form of the solution is easily seen to be y = c/(x2 −−1) and is
defined on any interval not containing either x = 1 or x = −1.
236
The solution of the DE is not f(x,y) = x2y −y. Rather it is f(x,y) = c; or if a constant
is used in the integration of g (y), we can then write the solution as ff(x,y) = 0.
Note, too, that the equation could be solved by separation of variables.
Example 7.7
Solve the following exact differential equation
equ
Solution:
In above equation we have
M (x,y) = e2y −y
− cosxy N (x,y) = 2xe2y + xcosxy+ 2y
we have then
Now
Integrating we have
which allows us
h’ (x) = 0
h(x) = c
therefore the solution has the following form
xe2y + sin xy+ y2 + c = 0
Example 7.8
Solve the initial value problem
Solution:
Now rewriting the differential equation we have
237
In above equation we have
we have then
Hence
So integrating we have
Now
Hence
we can rewrite
we have
c=3
An implicit solution of the differential equation is
238
7.3.3 Integrating Factors
In the last section the linear equation y + P(x)y = f(x)) can be converted into a
derivative when we multiply it with the integrating factor. The same idea can be
exact differential equation M(x,y)dx + N(x,y)dy=
used for a non-exact = 0. It is sometimes
possible to find an integrating factor ν(x,y) so that after multiplying,
lying, the left
left-hand
side of
ν(x,y
x,y) M (x,y)dx + ν(x,y)N(x,y)dy = 0 (7.18)
is an exact differential. The equation (7.18) is exact if and only if
(7.21)
( x−My)/M is a function of y,, then we can solve equation
in this case if and only if (N
(7.21). in that case we have
ν (y) = eʃ ((Nx−My)/M)dy (7.22)
Example 7.9
exact differential equation xydx+ (2x2 + 3y2 −20)dy= 0.
Solve the non-exact
239
Solution:
In above equation we have
we have then
Now the above equation is an exact differential equation simply follow the
previously mentioned examples to solve the exact differential equation we have
the family of solution as such
240
( 3 + αxy4 −2x)
14. Find the value of α so that the given differential equation is exact (y
2 2 3
dx+ (3xy + 20x y )dy dy= 0.
15. Show that given differential equation (−xy
( sin x+ 2y cos x) dx+(2x cos x) dy= 0
is not exact. Multiply the given differential equation by the integrating
factor ν(x,y) = xy and verify that the new equation is exact.
Solve the given differential equation by an appropriate integrating factor.
16. (2y2 + 3x) dx + 2xy
xy dy=
dy 0
17. 6xy dx + (4y + 9x2)dy= 0
18. (10 −6y + e−3x)dx−2dy dy = 0
19. Solve the differential equation xdx − (x2y + 4y) dy = 0 with initial-value
value y (4) = 0
by finding an appropriate integrating factor.
is called a Bernoulli differential equation where n is any real number other than 0
or 1. It is named after Jacob Bernoulli, who discussed it in 1695. Bernoulli
equations are special because they are nonlinear differential equations with known
exact solutions. A famous special case of the Bernoulli equation is the logistic
differential equation.
Example 7.10
Solve the following Bernoulli differential equation
241
Solution:
Rearranging the given differential equation we have the following form
In this case we have n = 2, so t = y1−2 = y−1 which gives y = t−1 hence we have
simplifying we have
We have
Now integrating
x−1t = −x + c
Simplifying we have
t = −x2 + cx
Since
t = y−1
hence the final solution is
Self-Assessment 7.3
Solve the Bernoulli differential equation by using an appropriate substitution.
242
This is a special case of the family of Lagrange equations,
First, we consider solving the more general Lagrange equation. Let in the
Lagrange equation, giving
y (x) = xf(q) + g (q)
Next, we differentiate with respect to x to find
(7.23)
We have introduced q = q(x),
q viewed as a function of x.. Let’s assume that we can
invert this function to find x = x(q).
). Then, from introductory calculus, we know
that the derivatives of a function and its inverse are related,
Simplifying we have
(7.24)
assuming that q −f (q) 0. We have converted the Lagrange equation into a first
order linear differential equation (7.24) for x(q).
). Using methods from earlier in the
chapter, we can in principle
nciple obtain a family of solutions.
Example 7.12
Solve the Lagrange equation y = 2xy’ −y’2.
Solution
We will start with Equation (7.24). Noting that
we have
243
Now putting the respective values
we have
This first order linear differential equation can be solved using an integrating
factor. Namely
Integrating we have
Simplifying we have
244
Taking q = y’, we have
y = xq+ g (q)
This is the Lagrange equation with f(q) = q.. Differentiating with respect to x, we
have
Rearranging, we find
x = −g’ (q)
So, we have the parametric solution
x = −g’ (q)
y = −qg’ (q) + g (q)
For the case that y = c,, it can be seen that y= cx + g(c)) is a general solution.
Example 7.13
Find the solution of the Clairaut equation y = xy’ −y’2.
Solutions;
As g (q) = −q2
There might also by a parametric solution not contained nthis
this family. It would be
given by the set of equations
x = −g’ (q) = 2q
245
Self Assessment 7.5
Solve the following differential equation.
1.
2.
3. y = qx−p2 −logq
4. (x −a)q2 + (x −y)q −y
− =0
2 3
5. y q −2xq + y = 0
6. x + yq= a + bq
7. q2 −6qx + 3y = 0
8. q2 = (q −1)y
Further Readings
1. Advanced Engineering Mathematics, Sixth edition Dennis G. Zill Loyola
Marymount University
246
Unit – 8
DIFFERENTIAL EQUATION OF
HIGHER ORDER
247
CONTENT
Introduction .................................................................................................249
Objectives ...................................................................................................249
8.1 Theory of Linear Equations ........................................................................250
8.1.1 Initial-Value and Boundary-Value Problems..................................250
8.1.2 Homogeneous Equations ...............................................................253
8.1.3 Differential Operators .....................................................................253
8.1.4 Linear Dependence and Linear Independence ................................255
8.1.5 Wronskian .......................................................................................257
8.1.6 Non-homogeneous Equations .........................................................259
8.1.7 Complementary Function ...............................................................260
8.2 Reduction of order ......................................................................................262
8.2.1 General Solution .............................................................................263
8.3 Homogeneous Linear Equations with Constant Coefficients ....................265
8.3.1 Auxiliary Equation .........................................................................266
8.4 Undetermined Coefficients ........................................................................272
8.4.1 Method of Undetermined Coefficients ..........................................272
8.5 Cauchy–Euler Equations ............................................................................279
8.5.1 Distinct Real Roots ........................................................................280
8.5.2 Repeated Real Roots .......................................................................281
8.5.3 Conjugate Complex Roots ..............................................................282
Further Readings .........................................................................................284
248
INTRODUCTION
In the previous unit, we have learnt the first order differential equations, whereas
in this unit we will learn about differential equations of higher order. In the first
section of this unit, we examine the underlying theory of linear differential
equations and methods for solving certain kinds off linear differential equations.
In the second section we will focus the methods of reduction of order of
equations. The difficulties that surround higher-order non linear differential
equations and the few methods that yield analytic solutions for such equations are
examined in the third section of the unit.
OBJECTIVES
After studying this unit, prospective teachers will be able to:
1. explain the concept of equations.
2. solve linear equations.
3. solve higher order homogeneous and non-homogeneous the linear
differential equations.
4. solve higher order linear differential equations with constant coefficients.
5. solve higher order linear differential equations using method of
undetermined coefficients.
6. describe Cauchy-Euler equations and solve them.
249
8.1 Theory of Linear Equations
We turn now to two or higher order differential equations. We will discuss some
of the underlying linear differential equations
e theory in this chapter.
8.1.1 Initial-Value
Value and Boundary-Value
Boundary Problems
Theorem 8.1
Example 1
The initial-value 3 ’’’ + 5y’’ −y’ + 7y = 0,
value problem 3y y (1) = 0, y’ (1) = 0,
y’’ (1) = 0
This equation has the trivial solution y = 0. Since the equation of the third order is
linear with constant coefficients, it follows that all the theorem (Existence of a
Unique Solution) conditions are met. So y = 0 is therefore the only ssolution
containing x = 1 at any interval.
Example 2
You should verify that the function y = 3e2x + e2x + 3x is a solution of the initial
initial-
value problem y’’’ −4y = 12x,
12 y (0) = 4, y’ (0) = 1.
250
Solution
The differential equation is linear, the coefficients as well as g(x)) = 12
12x are
continuous, and a2(x)) = 1 0 on any interval I containing x = 0. We conclude
from Theorem (Existence Solution) that the given function is the
Existence of a Unique Solution)
unique solution on I.
Boundary-Value
Value Problem
Another type of problem is to solve a two or greater linear differential equation of
order in which the dependent variable y or its derivatives are defined at different
points. A problem such as
The y (a) = y0 and y (b)) = y1 defined values are referred to as boundary conditions
(BC). A solution to the above problem is a function that satisfies the differential
equation at certain intervals I, containing a and b,, the graph of which passes
a,y0) and (b,y1).
through the two points (a,y
251
Other pairs of boundary conditions could be used for a second-order
second order differential
equation are
Where y0 and y1 indicate constants arbitrarily. These three conditions are only
special
ial cases of general boundary conditions
Example 3
00
Figure 8.2: The Boundary value problem Example 8.3 has many solutions.
252
If the boundary-value
value problem is changed to
x" + 16x
16 = 0 x(0) = 0, x(π/8) = 0
then x(0)
(0) = 0 still requires c1 = 0 in the solution. But applying x(π/8)
8) = 0 to x = c2
sin4t demands that 0 = c2 sin(π/2) → c2 = 0. Hence x = 0 is a solution of this new
boundary-value
value problem.
If the boundary-value
value problem is changed to
x" + 16x
16 = 0 x(0) = 0, x(π/2) = 1
then x(0)
(0) = 0 still requires c1 = 0 in the solution. But applying x(π/2)
2) = 0 to x = c2
sin4t demands that 1 = c2 sin2π → 1 = 0. Hence the boundary-value value problem has
no solution.
We will see that we must first be able to solve the corresponding homogeneous
equation in order to solve a non-homogeneous
non linear equation.(y = 0 is always a
linear homogeneous equation solution.)
Assumptions
the coefficients ai(x),i = 0,1,2,...,n,
0 are continuous;
the right-hand member g((x) is continuous; and
an(x) = 0 for every x in the interval.
In general
253
In general, we define a differential operator in the nth-order.
nth
n n−1
L = an (x)D + an−1 (xx)D + ......... + a1 (x)D + a0 (x) (8.5)
as a consequence of two essential differentiation properties
D [cf (x)] = cDf (x)
and
D [f (x) + g (x)] = Df (x) + Dg (x)
where c is constant.
can be written as
Superposition Principle
We see in the next theorem that the sum of two or more solutions of a
homogeneous linear differential equation, or superposition, is also a solution.
254
As L(y1 (x)) = 0 and L(y2 (x)) = 0, so
L[c1y1 (x) + c2y2 (x)] = 0
• A constant multiple y = c1y1(x) of a solution y1(x) of a homogeneous linear
differential equation is also a solution.
• A homogeneous linear differential equation always possesses the trivial
solution y = 0.
Example 4
A homogenous linear equation x3y000 −2xy0 + 4y = 0 on the interval (0,∞) has
solutions y1 = x2 and y2 = x2 lnx. By the superposition principle
y = c1x2 + c2x2 lnx
is also a solution of the given homogenous linear differential equation.
In the case of two functions f1(x) and f2(x), it is easy to understand these
definitions. If the functions are linearly dependent on an interval, there are
constants c1 and c2 which are not both zero in the interval c1f1(x)+c2f2 (x) = 0 for
every x. Hence if we consider c1 6= 0 it follows that
f1(x) = −c2f2 (x)/c1
If two functions are linearly dependent, then one is simply a constant multiple of
the other it
Conversely, if f1(x) = c2f2(x) for some constant c2, then at some interval (−1)f1(x) +
c2f2(x) = 0 for each x. Therefore, the functions are linearly dependent as at least
one of the constants (i.e., c1 = −1) is not zero. We conclude as follows:
Two functions are linearly independent when neither is a constant multiple of the
other on an interval.
For example, the functions f1 = (x)sin2x and f2 = (x)sinxcosx are linearly dependent
on (−∞,∞) because f1(x) is a constant multiple of f2(x). Recall from the double
255
angle formula for the sin that sin2x = 2sinxcosx. On the other hand, the functions
f1(x) = x and f2(x) = |x| are linearly independent on (−∞,∞).
Figure 8.3 and 8.4 shows that neither function is a constant multiple of the other
on the interval. It follows from the preceding discussion that the ratio f2(x)/f1(x) is
not a constant on an interval on which f1(x) and f2(x) are linearly independent.
The function such as f1 (x) = cos2 x,f2 (x) = sin2 x,f3 (x) = sec2 x and f4 (x) = tan2 x are
linearly dependent of (−π/2,π/2)/
f (x) = c1f1 (x) + c2f2 (x) + c3f3 (x) + c4f4 (x)
f (x) = c1 cos2 x + c2 sin2 x + c3 sec2 x + c4 tan2 x = 0
If c1 = c2 = 1,c3 = −1 and c4 = 1.
Set of functions f1(x),f2(x)...fn (x) are linearly dependent on I(interval) if one of the
function can be written as the linear combination of the others. Let there are three
functions f1(x),f2(x) and f3 (x) and at least one of these function is a linear
combination of the other two functions than we can say theses function are
linearly dependent on I Suc has
f3 (x) = c1f1 (x) + c2f2 (x)
When no function is a linear combination of the other functions, a set of n
functions is linearly independent on I.
256
8.1.5 Wronskian
where the primes denote derivatives, is called the Wronskian of the functions.
Thus, if we can show that W (y1,y2,...,yn) 6= 0 for some x0 in I,, then the solutions
y1,y2,...,yn are linearly independent on I. For example
257
Fundamental
undamental Set of Solutions
( 1,y2,...,yn) of the homogeneous linear nth
A set of linearly independent solutions (y nth-
order differential equation on an interval I is said to be a fundamental set of
solutions on the interval.
If we define
G(x) = C1y1(x) + C2y2(x),
we observe that G(x)) satisfies the differential equation, since it is a superposition
of two known solutions; G(x) satisfies the initial conditions
G(t) = C1y1(t) + C2y2(t) = k1
and
G’ (t) = C1y1(t) + C2y2(t) = k2
258
Y (x)) satisfies the same linear equation and the same initial conditions. Since the
solution of this linear initial-value
initial problem is unique (Theorem 8.1), ), we have Y
(x) = G(x) or Y (x) = C1y1(x) + C2y2(x).
Example 5
The homogenous linear differential equation y00−9y = 0 on the interval (−∞
−∞,∞) has
3x −3x
solutions y1 = e and y2 = e
Example 6
The homogenous linear differential equation from previous example y” −9y = 0
has general solutions of the following form y = 4sinh3x −5e−3x.
8.1.6 Non-homogeneous
homogeneous Equations
Any function yp free of arbitrary paramors that satisfies non homogenous linear
differential equation (8.4) is said to be a particular solution of the equation. For
example, it is a straightforward task to show that the constant function yp = 3 is a
particular solution of the nonhomogeneous equation y” + 9y = 27.
259
If you think about it, this makes sense, because the linear combination c1y1 (x) +
c2y2 (x),...ckyk (x) is mapped into 0 by the operator L = anDn + an−1Dn−1...a1D + a0,
whereas yp is mapped into g(x). If we use k = n linearly independent solutions of
the nth-order equation (8.3), then the expression in (??) becomes the general
solution of (8.4).
Proof Let L be the differential operator defined in (8.5), and let Y (x) and yp(x) be
particular solutions of the non-homogeneous equationL(y) = g(x). If we define
u(x) = Y (x) −yp(x), then by linearity of L we have
L(u) = L{Y (x) −yp(x)} = L(Y (x)) −L(yp(x)) = g(x) −g(x) = 0.
The linear combination yc(x) = c1y1 (x) + c2y2 (x),...cnyn (x), which is the general
solution of (8.3), is called the complementary function for equation (8.4). In other
words, in order to solve a non-homogeneous linear differential equation, we first
solve the associated homogeneous equation and then find any specific non-
homogeneous equation solution. The general solution of the nonhomogeneous
equation is then
y = complementary function + anyparticular solution
260
functions g1,g2,...,gk. That is, suppose ypi denotes a particular solution of the
corresponding differential equation
an (x)yn + an−1 (x)yn−1........ + a1 (x)y1 + a0y = gi (x)
where i = 1,2,...k. Then
yp (x) = yp1 (x) + yp2 (x) + .....ypk (x)
is a particular solution of
an (x)yn + an−1 (x)yn−−1........ + a1 (x)y1 + a0y = g1 (x) + g2 (x) + ..... + gk (x)
Proof:: We will show it for the case when k = 2. Let L be the differential operator
defined in (8.5), and let yp1
p (x)
and yp2(x)) be particular solutions of the non-homogeneous
non equations L(yy) = g1(x)
and L(y) = g2(x),
), respectively.
If we define
yp(x) = yp1(x) + yp2(x)
we want to show that yp is a particular solution of
L(y) = g1(x)g2(x).
The result follows again by the linearity of the operator L:
L(yp) = L{yp1(x)) + yp2(x)} = L(yp1(x)) + L(yp2(x)) = g1(x) + g2(x)..
4. Find an interval centered about x = 0 for which the given initial initial-value
problem has a unique solution (x −2)y +3y = x,.....y” (0) = 0,....y’ (0) = 11.
5. The given two-parameter
parameter family is a solution of the indicated differential
equation on the interval (−∞,∞).
( ∞). Determine whether a member of the family
can be found that satisfies the boundary conditions. y = c1ex cosx+ + c2ex sinx;
y” −2y’ +2y = 0 (a)yy (0) = 1,....y’ (π) = 0 (b)y (0) = 1,....y (π) = −1 ((c)y (0) =
1,....y (π/2) = 1 (d)yy (0) = 0,....y (π) = 1.
Determine whether the given set of functions is linearly dependent or
linearly independent on the interval (−∞,∞).
261
6. f1 (x) = x; f2 (x) = x2 f3 (x) = 4x −3x2
7. f1 (x) = 5; f2 (x) = x f3 (x) = ex
8. f1 (x) = x; f2 (x) = x −1 f3 (x) = x + 3
9. f1 (x) = 1 + x; f2 (x) = x f3 (x) = x2
Show that the given functions on the indicated interval form a fundamental
set of differential equation solutions. Form the general solution.
10. y” −y’ −12y = 0 e−33x,e4x,(−∞,∞)
11. y” −2y’ + 5y = 0 ex cos2x,e
cos2 x sin2x,(−∞,∞)
12. x2y” −6xy’ + 12y = 0 x3,x4,(0,∞)
3 ’’’ 2 ” ’
13. x y + 6x y + 4xy −4y − =0 x,x−2,x−2 lnx,(0,∞)
Show that the specified two-parameter
two parameter function family is the general
nonhomogeneous differential equation solution at the specified time.
Assume y(x)) is a known equation solution (8.7). We are looking for a second
solution y2(x)) of (8.7) so that at any interval I, y1 and y2 are linearly independent.
Remember that if y1 and y2 are linearly independent, their y2/y1 ratio on I is
nonconstant. That is
y2/y1 = u(x)
262
The idea is to find u(x)) by substituting y2(x) = u(x)y1(x)) into the given differential
equation. This method is called reduction of order since we must solve a first first-
order equation to find u.
Example 7
Given that y1 = ex is solution of y’’ −y = 0 on the interval (−∞,∞),
∞), use reduction of
order to find a second solution y2.
Solution
If y = u(x)y1(x) = u(x)ex, then the first two derivatives of y are obtained from the
product rule
Since ex 0, the last equation requires u’’ +2u’ = 0. If we make the substitution w =
u’, this linear second-order
order equation in u becomes w’ + 2w = 0, which is a linear
first-order equation in w.. Using the integrating factor e2x, we can write d/dx
d/dx[e2xw]
= 0. After integrating we get
w = c1e−2x
or
u0 = c1e−2x
Integrating again then yields
Hence
(8.8)
By choosing c2 = 0 and c1 = −2 we obtain the desired second solution, y2 = e−x.
Because W(ex,e−x) 6= 0 for every x, the solutions are linearly independent on ((-
∞,∞).
Suppose we divide by a2(x)( ) in order to put equation (8.7) in the standard form
=0 (8.9)
where P(x) and Q(x)) are continuous on some interval I.. Let us suppose further that
y1(x)) is a known solution of (8.9) on I and that y1(x)) 6= 0 for every x in the
interval. If we define y = u(x)y1(x), we have
263
This says that
so that requires
let u = w we have
(8.11)
It provides a good differentiation analysis to check that the function y2(xx) defined
in (8.11) matches equation (8.9) and that y1 and y2 are linearly independent at any
interval where y1(x)) is not zero.
Example 8
The function y1 = x2 is a solution of x2y” −3xy’ + 4y = 0. Find the general solution
on the interval (0,∞).
Solution
Rearranging the given differential equation in the standard form we have
264
From equation (8.11)
we have
265
8.3.1 Auxiliary Equation
It is obvious that the only way this exponential function can fulfill the differential
equation (8.13) is to select m as the root of the quadratic equation. Equation (8.14)
is known as auxiliary equation of the differential equation. The he two roots of
equation (8.14) are
The general solution of (8.12) will have three forms corresponding to the three
cases:
1. distinct.( 2 −4ac >0)
m1 and m2 are real and distinct.(b
2. equal.( 2 −4ac = 0)
m1 and m2 are real and equal.(b
3. numbers.( 2 −4ac <0)
m1 and m2 are conjugate complex numbers.(b
266
(8.16)
In (8.6) we have used the fact that −b/a = 2m1. The general solution is then
Example 9
Solve the following differential equation (a)y”
( −5y’ −3y = 0 (b)y’’ −10y’ +25
+25y =
’’ ’
0 (c)y + 4y + 7y = 0.
Solution
We give the auxiliary equations, the roots, and the corresponding general
solutions(y = emx)
(a)
m2 −5m −3 = (m −3)(2m + 1) = 0
So we have
267
(c)
m2 + 4m + 7 = 0
So we have
Example 10
4 ” + 4y’ + 17y = 0; y (0) = −1 y’ (0) = 2.
Solve the initial value problem 4y
Solution
We give the auxiliary equations, the roots, and the corresponding general
solutions(y = emx)
4m2 + 4m + 17 = 0
So we have
268
Hence the final solution is
For y" −k2y,, the auxiliary equation m2 −k2 = 0 has imaginary roots m1 = k and m2 =
−k.. From (8.15), the general solution of the differential equation is
y = c1ekx + c2e−kx
Higher-Order
Order Equations
In general, to solve an nth-order
nth differential equation
=0 (8.19)
where the ai, (i = 0,1,...,n,...,n)) are real constants, we must solve an nth
nth-degree
polynomial equation
anmn + an−1mn−1 + ........... + a2m2 + a1m + a0 = 0 (8.20)
If all the roots of (8.20) are real and distinct, then the general solution of (8.19) is
A fifth-degree
degree equation may have five distinct real roots, or three distinct real and
two complex roots, or one real and four complex roots, or five real but equal
roots, or five real roots, but two equal, and so forth. If m1 is a multiplicity k root of
an auxiliary equation of nth degree (i.e. k roots are equal to m1), linearly
independent solutions can be shown
269
Finally, it should be remembered that the coefficients of an auxiliary equation
always appear in conjugate pairs when they
they are real, complex roots. A cubic
polynomial equation can have a maximum of two complex roots.
Example 11
Solution
We give the auxiliary equations, the roots, and the corresponding general
solutions(y = emx)
So we have
m1 = 1 m2 = m3 = −2
Example 12
Solution
We give the auxiliary equations, the roots, and the corresponding general
solutions(y = emx)
So we have
m1 = m3 = i m2 = m4 = −i
The general solution have the following form
we can rewrite it
y = c1 cosx + c2 sinx + c3xcosx + c4xsinx
270
Self Assessment 8.3
Find the general solution of the differential equation given in the second order and
higher order.
271
8.4 Undetermined Coefficients
To solve a nonhomogeneous linear differential equation
We have to do two things: First is to find the complementary function yc; and
second a particular non-homogeneous
non equation solution yp. Then the general
solution on an interval I is y = yc + yp.
8.4.1 Method
ethod of Undetermined Coefficients
The first of two ways we will consider the method of undetermined coefficients to
obtain a particular solution yp. The underlying idea in this approach is a conjecture
about the yp form inspired by the kinds of functions that make up the gg(x) input
function, an educated guess actually. The general method is limited to non non-
homogeneous linear differential equations like (8.21) ai (i = 0,1,...n)) coefficients
are constants and where g(x)) is a constant, polynomial function, exponential
function eαx, sine or cosine function sinβx
sin or cosβx,, or finite sums and products of
such functions.
g(x) = k (a constant) is a polynomial function, strictly speaking. Since a constant
function is probably not the first thing to remember when thinking about
polynomial functions, we will continue to use the redundancy” constant functions,
polynomial functions” for emphasis.
Some examples of the types of inputs g(x)) suitable for this topic are the following
functions:
g (x) = 10
g (x) = x2 −5x
g (x) = 15x −6 + 8e−x
272
Where n is a non-negative
negative integer and α and β represent real numbers. The
undetermined coefficients method does not apply to equations (8.9) when
g (x) = lnx g (x) = 1/x
g (x) = tanx g (x) = sin−1 x
so on. In next section will consider differential equations in which the input g(x) is
a function of this last type.
The set of functions consisting of constants, polynomials, exponential eαx, sines,
and cosines has the spectacular property that their sums and products derivatives
are again sums and products of constants, polynomials, exponential eαx, sinsines, and
cosines.
Since the linear combination of derivatives a
a0yp must be the same as g(x), assuming that yp has the same form as g( g(x) seems
reasonable.
Example 13
Solve the following differential equation y” + 4y’ −2y = 2x2 −3x + 6 using the above
mentioned method of undetermined coefficients.
Solution
The first step is to solve the homogenous equation associated with the given
differential equation
In the second step the particular solution of the the function g(x)) which is a
quadratic polynomial, Let us assume solution to be a quadratic polynomial as well
yp = Ax2 + Bx + C
We seek to evaluate unique A, B, and C coefficients for which yp is a solution.
Replacing of yp and derivatives
273
into the given differential equation we have
Example 14
Solve the following differential equation to find the particular solution y”−y’+y =
2sin3x.
Solution
A natural first guess would be Asin3x for a particular solution. However, since
successive differentiations of sin3x
sin3 produce sin3x and cos3x,, instead we are
prompted to assume a specific solution that includes both of these terms
yp = Acos3x + B sin3x
The differentiation of yp and the replacing the results in the given differential
equation we have
274
y −y + y = −9Acos3x −9B
B sin3x −(−3Asin3x + 3B cos3x) + Acos3x + B sin3
sin3x
= (−9A −3B + A)cos3
)cos3x + (−9B + 3A + B)sin3x
(−8A −3B)cos3x + (−8B
( + 3A)sin3x = 2sin3x
Example 15
Solve the following differential equation y’’ −2 y’ −3y = 4x−5+6xe2x using the above
mentioned method of undetermined coefficients.
Solution
In the first step we have to find the solutionn of associated homogenous equa
equation
solution which is
yc = c1e−x + c2e3x
275
and
yp2 = Cxe2x + Ee2x
we have
yp = Ax + B + Cxe2x + Ee2x
Now putting the yp and its derivatives in the given differential equation we have
rearranging we have
2A + 3E))e2x + (−2A −3Ax −3B) = 4x −5 + 6xe2x
(−3Cx + (4C −2
Consequently
When examining the complementary function yc = c1ex + c2e4x, the problem here is
apparent. Remember that in yc, our assumption Aex already exists. This means that
ex is a solution of the associated homogeneous differential equation, and when
replaced with the differential equation a constant multiple Aex necessarily
produces zero.
276
So what should be the yp form? Inspired by Repeated Real Roots case let’s see if
we can find a specific form solution
yp = Axex
Now substituting
y’’p = Aex + Axex
y’p = 2Aex + Axex
we have
Case I
No function in the assumed particular solution is a solution of the associated
homogeneous differential equation
If g(x)) is sum of many say, m terms, then the assumption for a particular solution
yp consists of the sum of the trial forms yp1,yp2,.....ypm corresponding to these terms
y = yp1 + yp2 + ..... + ypm
Example 16
Determine the form of a particular solution of y” −9y’ +14y = 3x2 −5sin2x+8
+8xe6x.
Solution
The right-hand
hand side of the equation
g(x) = 3x2 −5sin2x + 8xe6x
consists of three different types of functions x2,sin2x and xe6x. The derivatives of
these functions yield, in turn, the additional functions x, 1; cos2x; and e6xx.
Therefore
For x2 we take
yp1 = Ax2 + Bx + C
277
For sin2x we take
yp2 = E cos2x + F sin2x
For xe6x we take
yp3 = Gxe6x + He6x
Note that none of the seven terms in this assumption for yp duplicates a term in the
complementary function yc = c1e2x + c2e7x.
Case II
A function in the assumed particular solution is also a solution of the associated
homogeneous differential equation.
Example 17
Find a particular solution of y’’ −2y’+ y = ex.
Solution
The complementary function is y = c1ex+c2xex, the assumption yp = Aex will fail
since it is apparent from yc that ex is a solution of the associated homogeneous
equation y’’ −2y’ + y = 0. Moreover, we will not be able to find a particular solution
of the form yp = Axex since the term xex is also duplicated in yc. We next try
yp = Ax2ex
substituting this in the given differential equation give us
simplifying we have
Hence we have
278
Self Assessment 8.4
Solve the following differential equation using undetermined coefficients method.
8.5 Cauchy–Euler
Euler Equations
The relative ease with which we were able to find explicit solutions in the
preceding sections of linear higher-order
higher differential
ifferential equations with constant
coefficients does not generally result in linear equations with variable coefficients.
It is an equation with variable coefficients, the overall solution of which can
always be expressed in terms of x, sines, cosines, logarithms
logarithms and exponentials.
Any linear differential equation of the form
279
where the coefficients an,an−1......a0 are constants, is known diversely as a Cauchy
Cauchy–
Euler equation oran Euler–Cauchy
Euler Cauchy equation. The differential equation is named in
honor of two of the most prolific mathematicians of all time, AugustinLouis
Cauchy (French, 1789– –1857) and Leonhard Euler (Swiss, 1707–1783). 1783). The
observable characteristic of this type of equation is that the degree k = n,n
0 of the monomial coefficients xk matches the order k of differentiation
−1,...,1,0
dky/dxk.
We shall begin the discussion by examining in detail the forms of the general
solutions of the homogeneous second-order
second equation
Depending on whether the roots of this quadratic equation are real and distinct,
real and equal, or complex, there are three different cases to be considered. The
roots appear in the last case as a conjugate pair.
Let m1 and m2 denote the real roots of (8.23) such that m1 6= m2. Then y1 = xm1 and y2
= xm2 form a fundamental set of solutions. Hence the general solution of (8.22) is
y = c1xm1 + c2xm2
280
Example
Solve x2y00 −2xy0 −4y = 0.
Solution
Rather than simply memorizing equation (8.23), it is better to conclude a few
times that y = xm is the solution to consider the origin and the discrepancy between
this new form of auxiliary equation. Twice differentiate,
we have
m1 = −1
m2 = 4
We get the general solution
y = c1x−1 + c2x4
If the roots of (8.23) are repeated (that is, m1 = m2), then we obtain only one
solution, namely, y = xm1. When the roots of the quadratic equation am2 +(+(b−a)m +
c = 0 are equal, the discriminant of the coefficients is necessarily zero. It follows
from the quadratic formula that the root must be m1 = −(b −a)/2a.
Now we can construct a second solution y2. We first write the Cauchy
Cauchy–Euler
equation in the standard form
281
Hence
Example 18
Solve 4
Solution
The substitution y = xm yields
when
(2m + 1)2 = 0
we have
m1 = m2 = −1/2
282
Nevertheless, as in the case of equations with constant coefficients, when the roots
of the auxiliary equation are complex, we want to write the solution only in terms
of real functions.
are also solutions. Since W(xα cos(β lnx),xα sin(β lnx)) = βx2α−1 6= 0, β > >0 on the
interval (0, ∞), we conclude that
y1 = xα cos(β lnx)
y2 = xα sin(β lnx)
constitute a fundamental set of real solutions of the differential equation.
Hence the general solution is
y = xα [c1 cos(β lnx) + c2 sin(β lnx)]
Example
Solve the initial-value
value problem 4 .
Solution
Taking y = xm we have
4x2m(m −1)xm−2 + 17xm = 0
Simplifying we have
283
Self Assessment 8.4
Solve the following differential equations using the above mention method.
Further Readings
1. Brummelen, Glen Van. Heavenly Mathematics: The Forgotten Art of
Spherical Trigonometry. Princeton: Princeton UP, 2013. Print.
284
Unit – 9
LAPLACE TRANSFORM
285
CONTENT
Introduction .................................................................................................287
Objectives ...................................................................................................287
9.1 Importance of Laplace Transform ..............................................................288
9.2 Laplace Transform ......................................................................................288
9.2.1 Definition..........................................................................................289
9.2.2 L Is a Linear Transform .................................................................292
9.2.3 Transforms of Some Basic Functions ..............................................293
9.2.4 Sufficient Conditions for Existence of L {f (t)} .............................293
9.2.5 Exponential Order ............................................................................294
9.3 The Inverse Laplace Transform ..................................................................297
9.3.1 Partial Fractions ................................................................................299
9.4 Transforms of Derivatives ..........................................................................300
9.5 Transforms of Derivatives ..........................................................................305
9.6 First shifting property on the s-axis ...........................................................310
9.7 Second shifting property on the t-axis ........................................................314
9.7.1 Inverse of L {f (t −a)U (t −a)} = e−asF (s) .......................................317
Further Readings .........................................................................................320
286
INTRODUCTION
In Mathematics, Laplace transform is powerful integral transform used too switch
a function from the time domain to s-domain. The Laplace transform can be used
in some cases to solve linear differential equations with given integral conditions.
So, keeping in view the importance of Laplace equation, we will discuss thi in the
first section of the unit. Transforms of some basic functions, exponential order,
the inverse Laplace transform, transforms of derivatives, methods of solving
learner differential equations using Laplace transform will be discussed in the
second section. The first shifting property on the s-axis and the second shifting
property on the t-axis and their uses have also been added in this unit.
OBJECTIVES
After studying this unit students will be able to understand the following concepts
• Laplace Transform
• Transforms of Some Basic Functions
• Exponential order The Inverse Laplace Transform
• Transforms of Derivatives
• Solving Linear Differential-Equations Using Laplace Transform
• Transforms of Derivatives
• First shifting property on the s-axis
• Second shifting property on the t-axis
287
9.1 Importance of Laplace Transform
The Laplace transformation, in mathematics, is an integral transformation named
Simon Laplace. This transforms a real variable t function
after its maker, Pierre-Simon
(often time) into a complex variable s function (complex frequency).
The transform has a lot of science and engineering applications. The transformation
of the Laplace is identical to the transformation of Fourier. While a function’s Fourier
transform
sform is a complex function of a real variable (frequency), a function’s Laplace
transform is a complex function of a complex variable. Laplace transformations are
usually limited to t with t = 0 functions.
A consequence of this restriction is that the variable
va s holomorphic function is the
Laplace transform of a function. In general, the Laplace transformation of a
distribution is a well-behaved
behaved function, unlike the Fourier transformation.
Complex variables methods may also be used to test Laplace transforms
transforms directly.
Complex variables methods can also be used to test Laplace transformations
directly. The Laplace transform has a representation of the power series as a
holomorphic function. This power series describes a function as a linear overlay
of function
ion moments. In probability theory, this approach has implications.
On a large class of functions, the Laplace transform is invertible. The inverse
transformation of Laplace takes a function of a complex variable s (often frequency)
and gives a function off a real variable t (often time). The Laplace transform
provides an alternative functional definition, providing a simple mathematical or
functional description of an input or output to a system, which often simplifies the
process of evaluating the system’s
system’s actions or synthesizing a new system based on a
set of specifications. For example, the transformation of Laplace from the time
domain into the frequency domain transforms differential equations into algebraic
equations and multiplication of convolution.
288
In addition, these two transforms possess the linearity property; this means a
linear combination of transforms is transforming a linear combination of
functions. For constants of α and β,
transforms a function f(tt) into a function of the variable s.. We are particularly
interested in integral transforms of this last kind, where the interval of integration
is the unbounded interval (0,∞).
(0
9.2.1 Definition
If f(t) is defined for t ≥ 0, then the improper integral K (s,t)f (t)dt is defined as
(9.1)
If the limit exists, it is said that the integral exists or is convergent; if the limit does
not exist, there is no integral and it is said to be divergent. In general, the
ntioned limit will exist for only certain variable s values. The choice of K(s,t)
aforementioned
across the board gives us an integral transformation that is particularly important.
(9.2)
is said to be the Laplace transform of f, provided the integral converges.
289
The transformation of the Laplace was most likely developed by Leonhard Euler
but is named after the famous French astronomer and mathematician Pierre
Pierre-Simon
Marquis de Laplace (1749–1827)
(1749 1827) who used the transformation in his work on
probability theory.
The result is a function of s when the integral (9.2) defining converges. In general
discussion, if we use a lower
lower case letter to denote the function being transformed,
the corresponding uppercase letter (lowercase) will be used to denote the
transformation of Laplace, for example.
Example 1
Evaluate L {1}.
Solution
As we know L hence
if s >0. When s >0, the exponent −sb is negative and e−sb → 0 as b → ∞. The
integral diverges for s <0.
0.
The use of the limit sign becomes a little tedious, so we are going to adopt the
notation as a shorthand to writing lim . For example
Example 2
Evaluate {t}.
Solution
As we know L hence
290
Using integration by parts
Example 3
Evaluate L {e−3t}.
Solution
Using Eq. (9.2) we have
Example 4
Evaluate L {e6t}.
Solution
Using Eq. (9.2) we have
L
Example 5
Evaluate L {sin2t}.
Solution
Using Eq. (9.2) we have
291
Using integration by parts we have
Now as L we have
Simplifying we have
Example 6
In this example, we use the results of the previous examples to illustrate the
Laplace transformation’s linearity
292
Similarly
Similarly
We state the generalization of some of the previous examples through the next
theorem. From this point on, we will also refrain throughout mentioning any
restrictions on s;; it is understood that s is sufficiently limited to ensure the
convergence of the correct
rrect Laplace transformation.
The integral that defines the Laplace transform does not have to converge. For
example, L {1/t} and L {et2} does not exist. Sufficient conditions guaranteeing
)} are that f be piecewise continuous on (0,∞)
the existence of L {f (t)} ∞) and that f be
of exponential order for t > T.
T A function f is piecewise continuous on (0 (0,∞) if, in
any interval defined by 0 ≤ a ≤ t ≤ b,, there are at most a finite number of points tk
293
(k = 1,2,...n(tk−1 < tk)), at which f has finite discontinuities and is continuous on
each open interval defined by tk−1 < t < tk.
Figure 9.1
9.2.5 Exponential Order
If f is an increasing function, then the condition |f| (t)| ≤ Mect, t > T simply states
that the graph of f on the interval (T,∞)
( ∞) does not grow faster than the graph of the
exponential function Mect, where c is a positive constant. The functions
f (t) = t f (t) = e−t f (t) = 2cost
are all of exponential order c = 1 for t >0 since we have,
294
exponential order since, as shown in the figure. et2 grows faster than any positive
linear power of e for t > c >0.
> This can also be written as
for any value of c.. For the similar justification we can say
e−stet2 → ∞ as t → ∞
for any s and so the improper integral words,{e t2}
diverges. In other words,
does not exist.
Theorem-Sufficient
Sufficient Conditions for Existence
If f(t)) is piecewise continuous on the interval (0,∞)
(0 ∞) and of exponential order, then
L {f (t)} exists for s > c.
Proof
By the additive interval property of definite integrals,
The integral I1 exists because it can be written as a sum of integrals over intervals
on which e−stf (t)) is continuous. Now f is of exponential order, so there exists
constants c, M >0, T >0 0 so that |f| (t)| ≤≤ Mect for t > T.. We can then write
Example 7
Evaluate L {f (t)} for f (tt) = {0, 0 ≤ t <32, t≥3.
Figure 9.3
295
Solution
continuous function appears in figure. Since f is defined in two
This piecewise-continuous
pieces, L {f (t)}
)} is expressed as the sum of two integrals
4.
5.
6. f (t) = et+7
7. f (t) = te4t
8. f (t) = e−t sin t
9. f (t) = t cos t
10. f (t) = 2t4
11. f (t) = 4t −10
296
12. f (t) = t2 + 6t −3
13. f (t) = (t + 1)3
14. f (t) = 1 + e4t
15. f (t) = (1 + e2t)2
16. f (t) = 4t2 −5 sin 3t
17. f (t) = sinh kt
18. f (t) = et sinh t
19. f (t) = sin 2t cos 2t
20. f (t) = sin(4t + 5)
While evaluating inverse laplace transforms, it often happens that a function under
consideration does not exactly match the form given in a table by a Laplace
transform F(s).). The function of s may need to be ”set” by multiplying and
dividing by a suitable constant.
Example 8
Evaluate L ?
297
Solution
From the above mentioned transformation L , so we have
Example 9
Evaluate L ?
Solution
From the transformation L so we have
L −1 Is a Linear Transform
The inverse Laplace transform is a linear transform,
Example 10
Evaluate L ?
Solution
We can write the given function in the following form
298
Example 11
Evaluate L ?
Solution
There exist unique constants A,B, and C such that
Since the denominators are the same the equality requires that numerators are also
same:
We have three unknowns and three equations, by solving the above set of
equation we have
hence we have
As we know L
299
Self Assessment 9.2
Find the inverse laplace transform of the following functions.
300
Doing integration by parts we have
Similarly,
It should be apparent from the results in (9.8), (9.9), and (9.10) that the Laplace
transforms the derivatives of a function f.. The next theorem gives the nth
derivative of f to the Laplace transform.
Theorem
If f’.......fn−1are continuous on (0,∞)
(0 ∞) and are of exponential order and if fn (t) is
piecewise continuous on (0,∞),
(0 then
L {fn (t)} = snF (s) −sn−1f (0) −sn−2f’ (0)..............sfn−2 (0) −fn−1 (0)
where as F (s) = L {f (t)} )}.
301
where the coefficients ai,((i = 0,1,...,n) and y0,y1,....yn−1 are constants.
If we solve the general transformed equation (9.11) for the symbol Y (s),
), we first
obtain P(s)Y (s) = Q(s)) + G(s), and then write
(9.7)
n n−1
where P(s) = ans + an−1s + ...... + a0, Q(s) is a polynomial in s of degree less
than or equal to n −1 1 consisting of the various products of the coefficients ai,(i =
), and the prescribed initial conditions y0,y1,....yn−1, and G(s)) is the Laplace
1,...,n),
transform of g(t). ). Typically we put the two terms in (9.12) over the least common
denominator and then decompose the expression into two or more partial
fractions. Finally, thee solution y(t) of the original initial-value
value problem is y(t) = L
{Y (s)},
)}, where the inverse transform is done term by term.
Example 12
Solution
Applying the laplace transformation to the given differential equation
302
As we know
Since the quadratic polynomial s2+4 does not factor using real numbers, its assumed
numerator in the partial fraction decomposition is a linear polynomial in s
Example 13
Solve y’’ −3y0 + 2y = e−4t where as y (0) = 1 and y’ (0) = 5.
Solution
Applying the laplace transformation to the given differential equation
303
As we know
The details of the decomposition of Y (s)) in into partial fractions have already been
carried out in previous example. After following the same procedure we have
Theorem-Behavior of F (s) as s → ∞
If f is piecewise continuous on (0,∞)
(0 ∞) and of exponential order, then limt→∞ L {f
(t)} =0.
Proof
Since f(t)) is piecewise continuous on the closed interval (0,∞),
(0 it is necessarily
bounded on the interval. That is,
|f (t)| ≤ M1 = M1e0t
Also, because f is assumed to be of exponential order, there exist constants γ,M2 >
0 and T >0 such that
|f (t)| ≤ M2eγt for t > T
If M denotes the maximum of {M
{ 1,M2} and c denotes the maximum of {0
{0,γ}, then
304
As a consequence of above Theorem we can say that functions of s such as F1(s) =
1 and F2(s) = s/(s + 1) are not the Laplace transforms of piecewise continuous
functions of exponential order since F1(s) 9 0 and F2 9 0(s) as s → ∞. But you
should not conclude from this that F1(s) and F2(s)) are not Laplace transforms.
There are other kinds of functions.
Similarly,
305
Now using equation (9.8) we have
It should be apparent from the results in (9.8), (9.9), and (9.10) that the Laplace
transforms the derivatives of a function f.. The next theorem gives the nth
derivative of f to the Laplace transform.
Theorem
If f ’.......fn−1are continuous on (0,∞)
(0 and are re of exponential order and if fn (t) is
piecewise continuous on (0,∞),
(0 then
L {fn (t)} = snF (s) −sn−1f (0) −sn−2f ’ (0)..............sfn−2 (0) −fn−1 (0)
where as F (s) = L {f (t)} )}.
306
(9.12)
n n−1
where P(s) = ans + an−1s + ...... + a0, Q(s) is a polynomial in s of degree less
than or equal to n −1 1 consisting of the various products of the coefficients ai,(i =
), and the prescribed initial conditions y0,y1,....yn−1, and G(s)) is the Laplace
1,...,n),
transform of g(t). ). Typically we put the two terms in (9.12) over the least common
denominator and then decompose the expression into two or more partial
fractions. Finally, thee solution y(t) of the original initial-value
value problem is y(t) = L
{Y (s)},
)}, where the inverse transform is done term by term.
Example14
Solution
Applying the laplace transformation to the given differential equation
As we know
Since the quadratic polynomial s2+4 does not factor using real numbers, its assumed
numerator in the partial fraction decomposition is a linear polynomial in s
307
Solving above linear equation we have A = 8,B = −2,C = 6. Hence
Example 15
Solve y’’ −3y’ + 2y = e−4t where as y (0) = 1 and y’ (0) = 5.
Solution
Applying the laplace transformation to the given differential equation
As we know
The details of the decomposition of Y (s)) in into partial fractions have already
been carried out in previous example. After following the same procedure we
have
308
Theorem
(Behavior of F (s) as s → ∞)
If f is piecewise continuous on (0,∞)
(0 ∞) and of exponential order, then limt→∞ L {f
(t)} = 0.
Proof
Since f(t)) is piecewise continuous on the closed interval (0,∞),
(0 it is necessarily
bounded on the interval. That is,
|f (t)| ≤ M1 = M1e0t
Also, because f is assumed to be of exponential order, there exist constants γ,M2 >
0 and T >0 such that
|f (t)| ≤ M2eγt for t > T
309
9.6 First Shifting Property on the s-axis
Evaluating transforms like L {e5tt3}, L {e2t cos2t} and L {e−2t sin2 sin2t} are
straightforward if we know L {t3}, L {cos2t} and L {sin2t}. }. Generally speaking,
if we know L {f (t)} = F (s),), it is possible to calculate the Laplace transformation
of an exponential multiple function f, i.e. L {eatf (t)}, )}, with out any additional
effort other than shifting F(s) to F(s −a).This This result is referred to as the first
theorem of shifting or the first theorem of translation.
Theorem
If L {f (t)} = F (s) and a is any real number
L
Proof
As we know
If we consider s a real variable, then the graph of F(s −a)) is the graph of F(s)
| If a >0, the graph of F(s)) is shifted a units
shifted on the s-axis by the amount |a|.
to the right, whereas if a <0,
< the graph is shifted |a| units to the left.
For emphasis it is sometimes useful to use the symbolism
where s → s −a means that in the Laplace transform F(s) of f(t)) we replace the
symbol s where it appears by s −a.
Example 16
Evaluate L {e5tt3}?
310
Solution
}
As we know L {eatf (t)}
)} = L {f (t) s→s−a ,hence
Example 17
Evaluate L {e−2t cos4t}?
Solution
As we know L {eatf (t)}
)} = L {f (t)} s→s−a ,hence
To compute the inverse of F(s−a) we must recognize F(s), find f(t)) by taking the
inverse Laplace transform of F(s), and then multiply f(t)) by the exponential function
eat. This procedure can be summarized symbolically in the following manner
−1
where f (t) = L {F (s)}.
)}.
Example 18
Evaluate L ?
Solution
Now through partial fraction decompositions, we have
311
Now putting the above transformations in the above equation we have
Example 19
Evaluate L ?
Solution
Now through partial fraction decompositions, we have
Example 20
Solve y″ −6y′ + 9y = t2e3t, y (0) = 2, y′ (0) = 17
312
Solution
Applying laplace transform on the given differential equation we have
As we know
simplifying we have
as we know
Now putting these in the above equation we have the solution of the given
differential equation
313
9.7 Second Shifting Property on the t-axis
Throughout technology, one often finds functions that are either “off” or “on”. For
example, after a period of time, an external force operating on a mechanical
system or a voltage exerted on a circuit can be shut off. Defining a special
function which is the number 0 (off) up to a certain time t a and then the number 1
(on) after that time is therefore convenient. This function is called the unit step
function or Heaviside function, named after Oliver Heaviside (1850–1925),
(1850 1925), the
renowned English electrical engineer, physicist, and mathematician.
The unit step function U (t −a) is defined to be
314
Figure 9.7: f(t) = 2 −3U (t −2) + U (t −3)
is the same as
f (t)) = g (t) −g (t) U (t −a) + h(t) U (t −a)
Similarly, a function of the type
can be written
f (t) = g (t)[U (t −a) −U (t −b)]
Example 21
Solution
We have a = 5, g (t) = 20tt and h(t) = 0, hence
f (t) = 20t −20tu (t −5)
315
As shown in figure, for a >0 > the graph of the function y = f(t −a)U (t −a)) coincides
with the graph of y = f(tt −a) for t ≥ a (which is the entire graph of y = ff(t), t ≥ 0,
shifted a units to the right on the t-axis) but is identically zero for 0 ≤ t ≤ a..
The exponential multiple of f(t) results in a translation of the transform F F(s) on the
axis. As a consequence of the next theorem we see that whenever F(s) is
t-axis.
ltiplied by an exponential function e−as, a >0,
multiplied 0, the inverse transform of the
product e−asF(s)) is the function f shifted along the t-axis
axis in the manner as shown in
figure. This result is called the second translation theorem or second shifting
theorem, which is presented next in its direct transform version.
as we know
So
316
Example 22
Evaluate L {U (t −a)}?
Solution
As we know L {f (t −a) U (t −a)} = e−asF (s)
Example 23
Evaluate L {2 −3 U (t −2)
2) + U (t −3)}?
Solution
U (t −a)} = e−asF (s)
As we know L {f (t −a)U
L
Example 24
Evaluate L ?
Solution
As we know L −1 {e−asF (s)} = f (t −a)U (t −a), so and
L −1 {F (s)} = e4t, so
317
Example 25
Evaluate L ?
Solution
As we know L
and
L −1 {F (s)} = cosh3t, so
Alternative form of
L {f (t −a) U (t −a)} = e−asF (s)
Example 26
π)}?
Evaluate L {cost U (t −π
Solution
( −a)} = e−as L {g (t + a)}, so a = π,g (t) = cost,
As we know L {g (t) U (t t, so
Example 27
Solve y′ + y = f (t), y (0) = 5,
5 where
?
Solution
As we can write f (t)) = 3cost
3cos U (t −π), so
318
Now using the partial fraction decomposition we can write
We
319
Using the laplace transform solve the following differential equations
Further Readings
1. Brummelen, Glen Van. Heavenly Mathematics: The Forgotten Art of
Spherical Trigonometry. Princeton: Princeton UP, 2013. Print.
320